Sie sind auf Seite 1von 67

Physician Assistant Clinical Knowledge Rating and Assessment Tool

1. Clinical Therapeutics/Hematology
A 6 year-old male presents with hemarthrosis o the let !nee. Coagulation studies re"eal the ollowing results#
PT 1$.% seconds &normal range 1$-1' seconds() *+R 1.,) aPTT %- seconds &normal range 1--$- seconds()
platelet count '.,),,,/microliter &normal range 1%,),,,-'%,),,,/microliter() and /leeding time ' minutes
&normal range $-1$ minutes(. 0hich o the ollowing is the /est treatment option or this patient1
A. 2esmopressin acetate
3. Corticosteroids
C. 4itamin K
2. Cryoprecipitate
&u( A. 2esmopressin acetate is indicated in "on 0ille/rand5s disease) which presents with a prolonged /leeding
time.
&u( 3. Corticosteroids are indicated in immune-mediated throm/ocytopenia.
&u( C. 4itamin K deiciency will prolong the PT greater than the aPTT. 4itamin K supplement is not indicated
in this patient.
&c( 2. Hemophilia A presents with a prolonged aPTT and normal platelet count and unction. Hemophilia A is
treated with actor 4*** concentrate or cryoprecipitate.
$. Clinical Therapeutics/*nectious 2iseases
An H*4 positi"e patient presents with worsening dementia) e"er) headache) and right hemiparesis. 6R* o the
/rain re"eals si7 lesions throughout the /rain that show ring enhancement and surrounding edema. 0hich o the
ollowing is the treatment o choice1
A. 8uladia9ine and pyrimethamine
3. Trimethoprim-sulametho7a9ole
C. Radiation therapy
2. 4entricular shunt placement
&c( A. To7oplasmosis is commonly noted in H*4 positi"e patients and presents with multiple ring-enhancing
lesions. Treatment o choice or possi/le to7oplasmosis is suladia9ine and pyrimethamine.
&u( 3. Trimethoprim-sulametho7a9ole is used or prophyla7is o to7oplasmosis) /ut not or treatment o acute
inection.
&u( C. Radiation therapy is indicated in C+8 lymphoma) which typically presents with a single lesion.
&u( 2. 8hunt placement is not indicated in patients with to7oplasmosis.
.. History : Physical/;/stetrics/<ynecology#
;n e7amination o a pregnant patient the physician assistant notes a /luish or purplish discoloration o the
"agina and cer"i7. This is called
A. Hegar5s sign.
3. 6c2onald5s sign.
C. Cullen5s sign
2. Chadwic!5s sign
&u( A. Hegar5s sign is the sotening o the cer"i7 that oten occurs with pregnancy.
&u( 3. 6c2onald5s sign is when the uterus /ecomes le7i/le at the uterocer"ical =unction at >-- wee!s.
&u( C. Cullen5s sign is a purplish discoloration perium/ilical and noted in pancreatitis.
&c( 2. Chadwic!5s sign is a /luish or purplish discoloration o the "agina and cer"i7.
'. 2iagnostic 8tudies/<astrointestinal/+utritional
A %$ year-old emale comes to the oice /ecause o /lac! stools or the past . days. 8he is ae/rile and she has
no pertinent physical e7amination a/normalities. 0hich o the ollowing is the most appropriate initial
diagnostic study1
A. 8tool or occult /lood
3. 8tool cultures
C. 8igmoidoscopy
2. A/dominal CT scan
&c( A. ;ccult /leeding) as e"idenced /y the patient5s history o /lac! stools) is initially "eriied /y a positi"e
ecal occult /lood test.
&u( 3. 8tool cultures are indicated in the e"aluation o acute diarrhea and not or the e"aluation o acute <*
/leeding.
&u( C. 6elena suggests a source o /leeding that is pro7imal to the ligament o Treit9) not a lower <* /leed.
8igmoidoscopy is used to e"aluate only lower <* /leeding sources.
&u( 2. A/dominal CT scan is indicated or e"aluation o o/scure /leeding in order to e7clude a pancreatic or
hepatic source o /leeding i endoscopy ails to identiy the source.
%. 1'-1. Health 6aintenance/2ermatology
0hich o the ollowing is considered a ris! actor or the de"elopment o malignant melanoma1
A. male gender
3. ina/ility to tan
C. ?apanese ethnicity
2. /rown-haired indi"iduals
&u( A. *ncidence o malignant melanoma is e@ual in males and emales.
&c( 3. *na/ility to tan and propensity to /urn are ris! actors or de"eloping malignant melanoma.
&u( C. 6alignant melanomas are most common in Caucasians and are rarely seen in the ?apanese population.
&u( 2. Red hair and rec!ling is one o the ma=or ris! actors or malignant melanoma) not /rown hair.
6. Clinical *nter"ention/+eurology
A '. year-old data entry cler! presents with a one-month history o pain and tingling in the right thum/) inde7
inger) and middle inger. Tinel5s sign and Phalen5s maneu"er are positi"e. The most appropriate inter"ention at
this time is
A. methylprednisolone &6edrol( dose pac!.
3. splint in neutral position.
C. o/ser"ation.
2. surgery.

&u( A. A 6edrol dose pac! will ha"e no aect on carpal tunnel syndrome.
&c( 3. 8plinting in neutral position relie"es impingement o the median ner"e) thus impro"ing symptoms o
carpal tunnel.
&u( C. ;/ser"ation will not impro"e symptoms.
&u( 2. 8urgical inter"ention is reser"ed or cases unresponsi"e to conser"ati"e therapy.
>. 0hich o the ollowing is a cause o prerenal a9otemia1
A. *nection
3. Renal to7ins
C. Poor renal perusion
2. Arinary tract o/struction
&u( A. *nection is associated with interstitial nephritis) which is considered a cause o intrinsic renal a9otemia.
&u( 3. This is one o the causes o intrinsic renal a9otemia.
&c( C. Renal hypoperusion is the cause o prerenal a9otemia) which may /e rapidly re"ersi/le when renal /lood
low and glomerular ultrailtration pressure are restored.
&u( 2. Arinary tract o/struction is the cause o postrenal a9otemia.
-. 8cientiic Concepts/Cardiology
0hich o the ollowing is the most common cause o secondary hypertension1
A. Renal parenchymal disease
3. Primary aldosteronism
C. ;ral contracepti"e use
2. Cushing5s syndrome
&c( A. Renal parenchymal disease is the most common cause o secondary hypertension.
&u( 3. Primary aldosteronism can cause secondary hypertension) /ut it is not the most common cause.
&u( C. ;ral contracepti"es can cause small increases in /lood pressure /ut considera/le increases are much less
common.
&u( 2. Cushing5s disease is a less common cause o secondary hypertension.
B. Health 6aintenance/<astrointestinal/+utritional
A new/orn weighs - pounds at /irth. ;n a"erage) what should the inant weigh at 1 year o age1
A. 16 pounds
3. $, pounds
C. $' pounds
2. $- pounds
&u( A. 8ee C or e7planation.
&u( 3. 8ee C or e7planation.
&c( C. An inant will triple /irth weight within the irst year o lie. A new/orn that weighs - pounds at /irth will
weigh appro7imately $' pounds at 1 year o age.
&u( 2. 8ee C or e7planation.
1,. History : Physical/C+T/;phthalmology
The most relia/le sign o acute otitis media &A;6( is
A. /ulging o the tympanic mem/rane.
3. loss o tympanic mem/rane mo/ility.
C. reddening o the tympanic mem/rane.
2. air /u//les /ehind the tympanic mem/rane.
&u( A. 3ulging and air /u//les /ehind the T6 represent ;6 with eusion.
&c( 3. Doss o tympanic mem/rane mo/ility during pneumoinsulation is the most relia/le sign or diagnosing
acute otitis media.
&u( C. Reddening o the eardrum is not relia/le as it may /e due to crying or other "ascular changes.
&u( 2. 8ee A or e7planation.
11. 2iagnosis/C+T/;phthalmology
0hich o the ollowing is a staphylococcal inection characteri9ed /y a locali9ed red swollen and acutely tender
a/scess o the upper or lower eyelid1
A. Hordeolum
3. A"eitis
C. Chala9ion
2. 2acryocystitis
&c( A. Hordeolum &stye( is a staphylococcal inection characteri9ed /y a locali9ed red swollen and acutely
tender a/scess o the upper or lower eyelid.
&u( 3. A"eitis is an intraocular inlammation in"ol"ing the u"eal tract.
&u( C. Chala9ion is a granulomatous inlammation o the mei/omian gland.
&u( 2. 2acryocystitis is an inection o the lacrimal sac due to o/struction o the nasolacrimal system.
1$. 8cientiic Concepts/;/stetrics/<ynecology
Progesterone inluence on the /reast tissue prior to menstruation causes
A. prolieration o the mammary ducts.
3. growth o the lo/ules and al"eoli.
C. prolieration o Cooper5s ligaments.
2. increase in the num/er o glands o 6ontgomery.
&u( A. Prolieration o the mammary ducts is under the inluence o estrogen.
&c( 3. <rowth o the lo/ules and al"eoli is under the inluence o progesterone. Prior to menses) the /reast
swelling that women notice is a result o the progesterone which is secreted rom the corpus luteum. 2uring
menses) the swelling su/sides.
&u( C) 2. 8ee 3 or e7planation.
1.. Clinical Therapeutics/Psychiatry/3eha"ioral 6edicine
A .6 year-old man has a ., pac!-year history o smo!ing cigarettes and wants to @uit. He is otherwise healthy
at this time. 0hich o the ollowing drugs would /e appropriate or him1
A. Amitriptyline &Cla"il(
3. 3upropion &0ell/utrin(
C. Eluo7etine &Pro9ac(
2. 4enlaa7ine &Ce7or(
&u( A) C) 2. 8ee 3 or e7planation.
&c( 3. The only two appro"ed drugs or aiding smo!ing cessation are nicotine and /upropion.
1'. History : Physical/;rthopedics/Rheumatology Topic# $ Author# Christine 3ruce
A patient presents with chronic /ac! pain. ;n physical e7amination testing) the patient is ound to ha"e
a/normalities o proprioception and "i/ration discrimination. 0hich o the ollowing portions o the spinal
column are most li!ely aected1
A. Dateral spinothalamic tract
3. 4entral spinothalamic tract
C. Posterior column
2. Transection o the cord
&u( A. The lateral spinothalamic tract aects pain and temperature sensation.
&u( 3. The "entral spinothalamic tract aects pressures and touch sensations.
&c( C. The posterior column aects proprioception &position sense( and "i/ration sense.
&u( 2. Patients with transection o the cord will ha"e loss o sensation distal to the area o in=ury along with
paralysis and hyperacti"e rele7es in the area distal to the transection.
1%. History : Physical/Cndocrinology
A '' year-old emale presents or ollow-up results o her lipid proile. 8he is asymptomatic and has a past
medical history o hypothyroidism treated with le"othyro7ine and hypertension controlled with atenolol
&Tenormin(. 8he drin!s an a"erage o 6 alcoholic /e"erages a day and smo!es 1 pac! per day or the last .$
years. Her amily history is unremar!a/le or premature coronary artery disease. Her asting /lood glucose is B-
mg/dD) total cholesterol is 1B- mg/dD) D2D cholesterol is 1.$ mg/dD) H2D cholesterol is %6 mg/dD and
triglycerides o B, mg/dD. C7cluding D2D cholesterol le"els) how many ma=or ris! actors or coronary artery
disease does this emale possess1
A. $
3. .
C. '
2. %
&c( A. This patient5s ma=or cardiac ris! actors are smo!ing and hypertension in addition to the ele"ated D2D
cholesterol.
&u( 3)C)2. 8ee A or e7planation.
16. 2iagnosis/*nectious 2iseases
A 6% year-old patient with steroid-dependent chronic o/structi"e lung disease presents with a headache that has
/een increasing in se"erity o"er the past wee!) accompanied /y nausea and "omiting. He denies e"er) /ut has
had photopho/ia and a sti nec!. 0hich o the ollowing is the most li!ely diagnosis1
A. Transient ischemic attac!
3. 3acterial meningitis
C. 6igraine headache
2. Cryptococcosis
&u( A. Transient ischemic attac!s present with ocal neurological indings rather than headaches.
&a( 3. 3acterial meningitis is typically acute in onset and causes e"er) /ut immunocompromised patients may
ha"e a slower onset and no e"er.
&u( C. 6igraines generally do not /egin in this age group) and are not accompanied /y nuchal rigidity.
&c( 2. Cryptococcus is an opportunistic ungal inection that aects immunocompromised patients) including
those with H*4) chronic steroid use) organ transplants) dia/etes mellitus) and chronic renal or li"er disease. The
most common clinical presentation is that o meningitisF e"er is present in only a/out hal o patients.
1>. 2iagnostic 8tudies/;/stetrics/<ynecology
A ', year-old emale presents with a 1.% cm well-circumscri/ed mass noted on mammography in the right
upper) outer @uadrant. 0hich o the ollowing procedures is most appropriate and should /e done ne7t1
A. Eine needle aspiration
3. 3RCA 1 and 3RCA $ genetic testing
C. 8erum CA-1$%
2. Radiation therapy
&c( A. Eine needle aspiration is airly accurate) easily perormed) and has minimal mor/idity.
&u( 3. Although 3RCA 1 and $ genetic tests are used in the ris! assessment or possi/le /reast and o"arian
cancer) it would not replace the need to perorm a more deiniti"e e"aluation o an identiied /reast mass.
&u( C. 8erum CA-1$% is a tumor mar!er or o"arian) not /reast) cancer.
&u( 2. Radiation therapy is only indicated ater a diagnosis o /reast cancer is pro"en and may /e used as
ad=uncti"e therapy.
1-. History : Physical/;/stetrics/<ynecology
;n e7amination o a pregnant patient the physician assistant notes the undal height is at the le"el o the
um/ilicus. This corresponds to what gestational age1
A. 16 wee!s
3. $, wee!s
C. $' wee!s
2. $- wee!s
&u( A) C) 2. 8ee 3 or e7planation.
&c( 3. At $,-$$ wee!s) the undal height is typically at the le"el o the um/ilicus.
1B. >'. History : Physical/Pulmonology
0hich o the ollowing is a common symptom associated with laryngotracheo/ronchitis &"iral croup(1
A. 2rooling
3. High e"er
C. GHot potatoG "oice
2. 3ar!ing cough
&u( A. 2rooling and a Ghot potatoG "oice are seen with epiglottitis) not "iral croup.
&u( 3. Ee"er is usually a/sent or low-grade in patients with "iral croup.
&u( C. 8ee A or e7planation.
&c( 2. 4iral croup is characteri9ed /y a history o upper respiratory tract symptoms ollowed /y onset o a
/ar!ing cough and stridor.
$,. 2iagnostic 8tudies/Pulmonology
A $. year-old emale with history o asthma or the past % years presents with complaints o increasing
shortness o /reath or $ days. Her asthma has /een well-controlled until $ days ago. 8ince yesterday) she has
/een using her al/uterol inhaler e"ery ' to 6 hours. 8he is normally "ery acti"e) howe"er yesterday she did not
complete her ., minute e7ercise routine due to increasing dyspnea. 8he denies any cough) e"er) recent
surgeries) or use o oral contracepti"es. ;n e7amination) you note the presence o prolonged e7piration and
diuse whee9ing. The remainder o the e7am is unremar!a/le. 0hich o the ollowing is the most appropriate
initial diagnostic e"aluation prior to initiation o treatment1
A. Chest 7-ray
3. 8putum gram stain
C. Pea! low
2. 4entilation-perusion scan
&u( A. A chest 7-ray should /e ordered in an asthmatic patient only i you are concerned a/out the presence o
pneumonia or pneumothora7) neither o which is supported /y the H:P indings noted a/o"e.
&u( 3. A sputum gram stain is perormed in patients who you suspect ha"e an inectious process) such as
pneumonia.
&c( C. A pea! low reading will help you to gauge her current e7tent o airlow o/struction and is helpul in
monitoring the eecti"eness o any treatment inter"entions.
&u( 2. A "entilation-perusion scan &4/H scan( is indicated in cases o suspected pulmonary em/olism. The
patient a/o"e does not ha"e any ris! actors that would lead you to suspect such a diagnosis.
$1. Health 6aintenance/Pulmonology
A . year-old girl is diagnosed with atopic dermatitis. 0hich o the ollowing disorders is this child at ris! or in
the uture1
A. Asthma
3. Tinea pedis
C. 8@uamous carcinoma
2. 8ystemic lupus erythematosus &8DC(
&c( A. Ap to %,I o patients with atopic dermatitis de"elop asthma and/or allergic rhinitis in the uture.
&u( 3. Patients with atopic dermatitis are more li!ely to get superimposed "iral or /acterial inections such as
herpes simple7 or staphylococcal) /ut they are not more at ris! or ungal inections.
&u( C. Patients with atopic dermatitis are at no greater ris! or any s!in cancer.
&u( 2. Dupus is a connecti"e tissue disorder o the immune system) /ut unrelated to atopic dermatitis.
$$. Clinical *nter"ention/*nectious 2iseases
A . year-old /oy is seen in the oice with a %-day history o e"er) erythema) edema o the hands and eet) a
generali9ed rash o"er the /ody) /ilateral con=uncti"al in=ections) issuring and erythema o the lips) and cer"ical
adenopathy. Antistreptolysin A &A8;( titer and throat culture are negati"e. The most serious systemic
complication associated with this disorder is
A. renal.
3. cardiac.
C. pulmonary.
2. hepatic.
&u( A. 8ee 3 or e7planation.
&c( 3. The patient most li!ely has Kawasa!i syndrome. The ma=or complication with this disorder is coronary
artery aneurysms) which are reported in up to $,I o aected children. The etiology o this disorder is
uncertain) although a /acterial to7in with super antigen properties may /e in"ol"ed.
&u( C. 8ee 3 or e7planation.
&u( 2. Children with Kawasa!i syndrome may ha"e associated hydrops o the gall/ladder) /ut li"er in"ol"ement
is not part o this disorder.
$.. Clinical Therapeutics/Cardiology
0hich o the ollowing medication classes is the treatment o choice in a patient with "ariant or Prin9metal5s
angina1
A. Calcium channel /loc!ers
3. ACC inhi/itors
C. 3eta /loc!ers
2. Angiotensin ** receptor /loc!ers
&c( A. Calcium channel /loc!ers are eecti"e prophylactically to treat coronary "asospasm associated with
"ariant or Prin9metal5s angina.
&u( 3. ACC inhi/itors are not a treatment or coronary "asospasm.
&h( C. 3eta /loc!ers ha"e /een noted to e7acer/ate coronary "asospasm potentially leading to worsening
ischemia.
&u( 2. Angiotensin ** receptor /loc!ers are not a treatment or coronary "asospasm.
$'. Clinical Therapeutics/;/stetrics/<ynecology
Pharmacologic treatment o a patient with gestational dia/etes should consist o which o the ollowing1
A.;ral hypoglycemic agents
3. Regular insulin
C. ;ral corticosteroids
2. <lucagon
&h( A. ;ral hypoglycemic agents ha"e no role in the treatment o gestational dia/etes as these drugs may cross
the placenta and harm the etus.
&c( 3. Regular insulin is the drug o choice as this will maintain the mother5s /lood sugar /ut not cross the
placenta.
&h( C. ;ral corticosteroids ha"e no role in the treatment o gestational dia/etes. Corticosteroids will cause the
/lood glucose to increase.
&u( 2. <lucagon is gi"en to patients when their /lood glucose is a/normally low. <lucagon stimulate
gluconeogenesis.
$%. Health 6aintenance/<astrointestinal/+utritional
0hich o the ollowing is an indication or "accination against hepatitis A1
A. *llicit drug users
3. Health care wor!ers
C. Renal dialysis patients
2. Routine "accination starting at /irth
&c( A. Hepatitis A "accine is recommended or illicit drug users) anyone li"ing or tra"eling to endemic areas)
sewage wor!ers) ood handlers) homose7ual and /ise7ual men) animal handlers) patients with a history o
chronic li"er disease or a clotting actor disease as well as children and wor!ers in day care settings and
institutions.
&u( 3. Health care wor!ers) renal dialysis patients and routine "accination starting at /irth are some o the
recommendations or "accination against hepatitis 3) not hepatitis A.
&u( C) 2. 8ee 3 or e7planation.
$6. History : Physical/Psychiatry/3eha"ioral 6edicine
The 286-*4 classiies mental disorders /y using i"e a7es in completing the process. A7is *** is used to
identiy which o the ollowing1
A. Clinical disorders and other conditions that may /e the ocus o clinical attention
3. Any physical disorder or general medical condition that is present in addition to the mental disorder
C. The psychosocial and en"ironmental pro/lems that ha"e had a signiicant contri/ution to the de"elopment or
e7acer/ation o the disorder
2. Personality disorders and/or mental retardation
&u( A. A7is * identiies clinical disorders and other conditions that may /e the ocus o clinical attention.
&c( 3. A7is *** identiies any physical disorder or general medical condition that is present in addition to the
mental disorder.
&u( C. A7is *4 identiies the psychosocial and en"ironmental pro/lems ha"ing a signiicant contri/ution to the
disorder.
&u( 2. A7is ** identiies personality disorders and mental retardation.
$>. 2iagnostic 8tudies/C+T/;phthalmology
A $ month-old inant presents or a routine health maintenance "isit. The mother has /een concerned a/out the
inant5s hearing since /irth. Physical e7amination re"eals no apparent response to a sudden loud sound. 0hich
o the ollowing is the most appropriate diagnostic e"aluation1
A. audiometry
3. tympanometry
C. acoustic relectometry
2. auditory-e"o!ed potentials
&u( A. Pure tone audiometry can /e used to screen or hearing deicits in children o"er the age o . years.
&u( 3. Tympanometry is used to identiy an eusion as the cause o hearing loss) /ut in inants o"er the age o
months.
&u( C. Acoustic relectometry measures the spectral gradient o the tympanic mem/rane) /ut is not used
clinically due to concerns a/out its relia/ility.
&c( 2. 3rainstem auditory-e"o!ed potentials e"aluate the sensory pathway and identiy the site o any
anatomical disruption. The test does not re@uire any acti"e response rom the patient and is useul in the
e"aluation o suspected hearing loss in an inant.
$-. 2iagnosis/Pulmonology
A %. year-old emale status post a/dominal hysterectomy . days ago suddenly de"elops pleuritic chest pain and
dyspnea. ;n e7am) she is tachycardic and tachypneic with rales in the let lower lo/e. A chest 7-ray is
unremar!a/le and an CK< re"eals sinus tachycardia. 0hich o the ollowing is the most li!ely diagnosis1
A. Atelectasis
3. Pneumothora7
C. Pulmonary em/olism
2. 6yocardial inarction
&u( A. 8mall atelectasis is commonly asymptomatic) while large atelectasis may produce signs o dyspnea and
cough. C7am re"eals a/sence o /reath sounds in the area in"ol"ed and dullness to percussion. A chest 7-ray
would re"eal "arious indings dependent on the location o the atelectasis) /ut would not /e normal.
&u( 3. 0hile a pneumothora7 commonly presents with pleuritic chest pain and dyspnea) e7am would re"eal the
presence o diminished /reath sounds and hyperresonance on the in"ol"ed side. A chest 7-ray would re"eal
presence o a pleural line on the e7piratory chest 7-ray.
&c( C. This patient5s ris! actors or pulmonary em/olism include ad"anced age) surgery) and prolonged /edrest.
0hile the diagnosis o pulmonary em/olism is diicult to ma!e due to nonspeciic clinical indings) the most
common symptoms include pleuritic chest pain and dyspnea associated with tachypnea. Chest 7-ray and CK<
are usually normal.
&u( 2. 0hile a myocardial inarction usually presents with dyspnea) the chest pain is not usually pleuritic in
nature. An CK< would commonly re"eal 8T segment changes which would /e consistent with ischemia or
inarct.
$B. Health 6aintenance/Pulmonology
0hich o the ollowing is an independent ris! actor or de"elopment o a mesothelioma1
A. Cigarette smo!ing
3. As/estos e7posure
C. Radon gas e7posure
2. Chronic o/structi"e lung disease
&u( A. There has not /een any e"idence o association /etween cigarette smo!ing and the de"elopment o
mesothelioma.
&c( 3. 8tudies conirm the association o as/estos e7posure to the de"elopment o mesothelioma.
&u( C. Ater cigarette smo!ing) radon gas is the second most common ris! actor or de"elopment o
/ronchogenic lung cancer) not mesothelioma.
&u( 2. Chronic o/structi"e lung disease is associated with an increased ris! o /ronchogenic lung cancer) not
mesothelioma.
.,. Clinical Therapeutics/Pulmonology
A .' year-old emale with a history o asthma presents with complaints o increasing asthma attac!s. The
patient states she has /een well-controlled on al/uterol inhaler until one month ago. 8ince that time she notices
that she has had to use her inhaler .-' times a wee! and also has had increasing nighttime use a"eraging a/out
three episodes in the past month. 8pirometry re"eals greater than -%I predicted "alue. 0hich o the ollowing
is the most appropriate inter"ention at this time1
A. ;ral prednisone
3. ;ral theophylline &Theo-2ur(
C. 8almeterol &8ere"ent( inhaler
2. 3eclomethasone &H"ar(inhaler
&u( A. ;ral corticosteroids) such as prednisone) are added to therapy in se"ere persistent asthma. 0hile a course
o oral corticosteroids may /e needed or mild e7acer/ations o asthma) they are not added until inhaled
corticosteroids ha"e ailed to control the symptoms.
&u( 3. 2ue to its saety proile) oral theophylline is now considered a third or ourth line treatment option or
asthma.
&u( C. Dong acting inhaled /eta$-agonists) such as salmeterol) are not added to the treatment regimen until the
symptoms indicate a moderate persistent asthma. Dong acting inhaled /eta$-agonists should also not /e used in
place o inhaled steroids.
&c( 2. This patient has progressed to mild persistent asthma. *n addition to her inhaled /eta$-agonist &al/uterol()
she should /e started on an anti-inlammatory agent. *nhaled corticosteroids) such as /eclomethasone) are
preerred or long-term control.
.1. 8cientiic Concepts/Pulmonology
0hich o the ollowing mechanisms leads to a primary pneumothora71
A. Penetrating or /lunt trauma orces
3. Anderlying lung cancer
C. Pressure o air in the pleural space e7ceeds room air pressure
2. Rupture o su/pleural apical /le/s due to high negati"e intrapleural pressures
&u( A. Penetrating or /lunt trauma orce is responsi/le or a traumatic pneumothora7.
&u( 3. A pneumothora7 that results rom an underlying lung disease is classiied as a secondary pneumothora7.
&u( C. 0hen pressure o air in the pleural space e7ceeds room air pressure) it leads to a tension pneumothora7.
&c( 2. A primary spontaneous pneumothora7 is thought to result rom a rupture o su/pleural apical /le/s
secondary to high negati"e intrapleural pressures.
.$. Clinical Therapeutics/;/stetrics/<ynecology
0hat is the initial treatment o choice or hyperthyroidism in a 1,-wee! pregnant patient1
A. +o treatment is necessary.
3. Propylthiouracil &PTA(
C. Radioiodine treatment
2. 8u/total thyroidectomy

&h( A. Although thyroid unction tests are altered in pregnancy true hyperthyroidism can occur and should /e
treated.
&c( 3. This is the initial treatment o choice.
&h( C. Radioiodine treatment is contraindicated in pregnancy.
&u( 2. 8u/total thyroidectomy is an option or pregnant patients during the second or third trimesters. 8urgery is
not indicated or irst-trimester pregnancies.
... 2iagnosis/C+T/;phthalmology
A patient presents with eye pain and /lurred "ision. 8nellen testing re"eals "ision o $,/$,, in the aected eye
and $,/$, in the unaected eye. Eluorescein staining re"eals the presence o a dendritic ulcer. 0hich o the
ollowing is the most li!ely diagnosis1
A. 4iral !eratitis
3. Eungal corneal ulcer
C. Acanthamoe/a !eratitis
2. 3acterial corneal ulcer
&c( A. Herpes 8imple7 "irus is a common cause o dendritic ulceration noted on luorescein staining.
&u( 3. Eungal corneal ulcers ha"e an indolent course with intraocular inection /eing common /ut luorescein
staining is negati"e or a dendritic pattern.
&u( C. Acanthamoe/a !eratitis has a wa7ing and waning course o"er se"eral months and has no luorescein
staining in a dendritic pattern.
&u( 2. 3acterial corneal ulcers can progress aggressi"ely resulting in corneal peroration. Eluorescein staining
does not occur in a dendritic pattern.
.'. Clinical Therapeutics/Cardiology
A 6. year-old emale with history o dia/etes mellitus presents or /lood pressure ollow-up. At her last two
"isits her /lood pressure was 1%,/B$ and 1%$/B6. Today in the oice her /lood pressure is 1'6/B$. Recent /lood
wor! shows a 8odium 1', mC@/D) Potassium '.$ mC@/D) 3A+ o $. mg/dD) and Creatinine o 1.1 mg/dD.
0hich o the ollowing is the most appropriate initial medication in this patient1
A. Tera9osin &Hytrin(
3. Atenolol &Tenormin(
C. Disinopril &Jestril(
2. Hydrochlorothia9ide &HCTJ(
&u( A. Alpha /loc!ers are not the treatment o choice in a dia/etic with hypertension.
&u( 3. Patients with hypertension and dia/etes may re@uire a 3eta /loc!er) /ut it should /e added to an ACC
inhi/itor i the ACC inhi/itor is ineecti"e on its own.
&c( C. ACC inhi/itors should /e part o the initial treatment o hypertension in dia/etics /ecause o /eneicial
eects in dia/etic nephropathy and is the most appropriate initial medication.
&u( 2. Patients with hypertension and dia/etes mellitus may re@uire a diuretic) /ut it should /e added to an ACC
inhi/itor i the ACC inhi/itor is ineecti"e on its own.
.%. 2iagnostic 8tudies/Cardiology
0hat is the CK< maniestation o cardiac end-organ damage due to hypertension1
A. Right /undle /ranch /loc!
3. Det "entricular hypertrophy
C. Right "entricular hypertrophy
2. 8T segment ele"ation in lateral precordial leads
&u( A. Right /undle /ranch /loc! is caused /y a delay in the conduction system in the right "entricle. *t may /e
caused /y right "entricular hypertrophy or conditions with higher pulmonic resistance such as cor pulmonale.
Hypertension) howe"er) is li!ely to cause changes in the let "entricle rather than the right "entricle.
&c( 3. Dong-standing HT+ can lead to let "entricular hypertrophy with characteristic changes noted on CK<.
&u( C. 8ee A or e7planation.
&u( 2. 8T segment ele"ation is a sign o acute myocardial inarction not hypertension.
.6. Health 6aintenance/Cardiology
Annual /lood pressure determinations should /e o/tained /eginning at the age o
A. . years.
3. % years.
C. 1$ years.
2. 1- years.
&c( A. Periodic measurements o /lood pressure should /e part o routine pre"enti"e health assessments
/eginning at the age o . years.
&u( 3) C) 2. 8ee A or e7planation.
.>. 8cientiic Concepts/;rthopedics/Rheumatology T
*n adults and intra"enous drug a/users) which o the ollowing /ones is most commonly aected with acute
osteomyelitis1
A. Eemur
3. Humerus
C. 4erte/ral spine
2. Ti/ia
&u( A. Dong /ones are most commonly aected with osteomyelitis in children.
&u( 3. 8ee A or e7planation.
&c( C. The /ones o the "erte/ral spine are most commonly aected in a patient with osteomyelitis. ;rganisms
reach the well-perused "erte/ral /ody o adults "ia spinal arteries and @uic!ly spread rom the end plate into
the dis! space and then to the ad=acent "erte/ral /ody. The inection may originate in the urinary tract and
intra"enous drug use carries an increased ris! o spinal inection
&u( 2. 8ee A or e7planation.
.-. Clinical Therapeutics/;/stetrics/<ynecology
Treatment o the patient with Pediculosis pu/is consists o which o the ollowing1
A. Permethrin &+i7( cream
3. Clotrima9ole &<yne-Dotrimin(
C. Podoilo7 &Condylo7( solution
2. 8elenium sulide &8elsun( suspension
&c( A. Permethrin 1I cream/shampoo is used to !ill the louse and remo"e the eggs rom the hair shats.
&u( 3. Clotrima9ole is an antiungal agent and is not used to treat parasitic inestation.
&u( C. Podoilo7 ,.%I solution is used to treat Condyloma accuminata.
&u( 2. 8elenium sulide suspension is used to treat Tinea "ersicolor ungal inection.
.B. 2iagnostic 8tudies/Cndocrinology
A '. year-old asymptomatic dia/etic emale is ound to ha"e an ele"ated total calcium le"el o 1$.' mg/dD.
0hich o the ollowing tests must /e assessed in order to e"aluate this la/oratory a/normality1
A. *ntact parathyroid hormone
3. 8erum al/umin
C. $' hour urine calcium le"el
2. Complete /lood count
&u( A. *ntact parathyroid hormone le"els are only o/tained or patients with true hypercalcemia with an
un!nown etiology.
&c( 3. 8ince appro7imately %,I o calcium is protein /ound) total calcium le"els should /e interpreted relati"e
to al/umin le"els.
&u( C. 2emonstration o e7cessi"e calcium in the urine does not pro"ide any additional inormation regarding
the increased serum calcium.
&u( 2. Complete /lood count has no relationship to the serum calcium le"els.
',. History : Physical/Cardiology
0hich o the ollowing conditions would cause a positi"e Kussmaul5s sign on physical e7amination1
A. Det "entricular ailure
3. Pulmonary edema
C. Coarctation o the aorta
2. Constricti"e pericarditis
&u( A. Det "entricular ailure results in the /ac!-up o /lood into the let atrium and then the pulmonary system
so it would not /e associated with Kussmaul5s sign.
&u( 3. Pulmonary edema primarily results in increased pulmonary pressures rather than ha"ing eects on the
"enous inlow into the heart.
&u( C. Coarctation o the aorta primarily aects outlow rom the heart due to the stenosis resulting in delayed
and decreased emoral pulsesF it has no eect on causing Kussmaul5s sign.
&c( 2. Kussmaul5s sign is an increase rather than the normal decrease in the C4P during inspiration. *t is most
oten caused /y se"ere right-sided heart ailureF it is a re@uent inding in patients with constricti"e pericarditis
or right "entricular inarction.
'1. Clinical *nter"ention/+eurology
Treatment o 3ell5s palsy includes which o the ollowing1
A. Acyclo"ir
3. Reassurance o the patient5s reco"ery
C. Reerral to a neurosurgeon
2. Clectromyography
&c( 3. 3ell5s palsy is a peripheral neuropathy o cranial ner"e 4**. Although it has /een suggested it may /e
related to an acti"ation o herpes simple7 "irus) there is little empiric e"idence or this. Appro7imately 6,I o
cases o 3ell5s palsy reco"er without treatment and patient reassurance o this is ad"ised. Clectromyography
may pro"ide aid in the prognosis) /ut not as a treatment option. A neurosurgeon has no role in the management
o 3ell5s palsy.
&u( A) C) 2. 8ee 3 or e7planation.
'$. 2iagnostic 8tudies/C+T/;phthalmology
2uring a /ase/all game) a $$ year-old college student is hit in the right eye /y a /ase/all. He complains o
/lurry "ision in that eye. ;n physical e7am) the physician assistant notes proptosis o the right eye) and
limitation o mo"ement in all directions. ;n CT scan) which o the ollowing is most li!ely to /e seen1
A. Eracture o the medial or/ital wall
3. Prolapse o or/ital sot tissue
C. Hematoma o the or/it
2. ;r/ital emphysema
&u( A. Eracture o the medial or/ital wall is associated with diplopia rom medial rectus impingement) or/ital
emphysema and epista7is.
&u( 3. Prolapse o or/ital sot tissue) including inerior rectus muscle) inerior o/li@ue muscle) or/ital at) and
connecti"e tissue results in enophthalmos) ptosis) diplopia) anesthesia o the ipsilateral chee! and upper lip) and
limitation o upward ga9e and is seen with ractures o the or/ital loor.
&c( C. ;r/ital hemorrhage into the space surrounding the glo/e ollowing /lunt trauma and rupture o the or/ital
"essels results in increased ocular pressure) proptosis) "isual loss) and limitation o mo"ement in all direction.
CT re"eals a hematoma.
&u( 2. ;r/ital emphysema is seen with ractures o the medial or/ital wall or loor o the or/it into the ma7illary
and ethmoid sinuses respecti"ely. *t will not lead to proptosis.
'.. History : Physical/Cardiology
0hich o the ollowing physical indings is suggesti"e o atrial septal deect1
A. Ei7ed split 8$
3. *ncreased pulse pressure
C. Continuous mechanical murmur
2. 2ierence in /lood pressure /etween the let and right arm
&c( A. An atrial septal deect will cause a shunt o /lood rom the let to the right atrium. This will result in an
e@uali9ation in the amount o /lood entering /oth the let and right "entricles which eecti"ely eliminates the
normally wide splitting that inspiration typically causes in hearts without an atrial septal deect.
&u( 3. Pulse pressures relect the dierence in aortic and let "entricular "olumes that occur during "entricular
systole. *ncreased pulse pressures are seen in aortic regurgitation which is a dierent entity than atrial septal
deect.
&u( C. Continuous mechanical murmurs are noted in patients with patent ductus arteriosus.
&u( 2. 2ierences in /lood pressure /etween the let and right arms are seen in conditions such as coarctation o
the aorta.
''. 2iagnostic 8tudies/Pulmonology
0hich o the ollowing is essential to ma!e a diagnosis o cystic i/rosis1
A. Positi"e amily history
3. Cle"ated sweat chloride
C. Recurrent respiratory inections
2. Cle"ated trypsinogen le"els
&u( A. Cystic i/rosis is a genetic disease) /ut a positi"e amily history in and o itsel is not enough to diagnose
the condition.
&c( 3. The diagnosis o cystic i/rosis is made only ater an ele"ated sweat chloride test or demonstration o a
genotype consistent with cystic i/rosis.
&u( C. 0hile recurrent respiratory inections is a classic presentation o cystic i/rosis) the diagnosis relies on
conirmation) as noted in e7planation 3.
&u( 2. Trypsinogen le"els are used as a neonatal screening test and i ele"ated should /e ollowed /y more
deiniti"e testing to conirm the diagnosis.
'%. History : Physical/C+T/;phthalmology
*n inants) the eyes should mo"e in parallel without de"iation /y the age o
A. $ wee!s.
3. . months.
C. 6 months.
2. 1 year.
&u( A) 3) 2. 8ee C or e7planation.
&c( C. *ntermittent alternating con"ergent stra/ismus is re@uently noted or the irst 6 months o lie) /ut
reerral is indicated i it persists /eyond 6 months.
'6. History : Physical/Pulmonology
0hich o the ollowing physical e7am indings is consistent with moderate emphysema1
A. *ncreased tactile remitus
3. 2ullness to percussion
C. 2istant heart sounds
2. 2e"iated trachea
&u( A. Physical e7amination indings in emphysema include a midline trachea) diuse hyperresonant to
percussion) and decreased tactile remitus.
&u( 3) 2. 8ee A or e7planation.
&c( C. 2istant heart sounds are common in emphysema patients due to hyperinlation o the lungs.
'>. Clinical *nter"ention/<astrointestinal/+utritional
0hich o the ollowing is the most common indication or operati"e inter"ention in patients with chronic
pancreatitis1
A. 0eight loss
3. *ntracta/le pain
C. C7ocrine deiciency
2. To decrease ris! o cancer
&u( A. 0hile weight loss is common with chronic pancreatitis) it is not an indication or surgical inter"ention.
&c( 3. *ndications or surgical treatment o chronic pancreatitis include se"ere pain that limits the patient5s
unctioning or intracta/le pain despite the use o non-narcotic analgesics and a/sence o alcohol inta!e.
&u( C. 0hile the ma=ority o patients go on to de"elop 26 $% yrs ater the clinical onset o chronic pancreatitis)
this is not an indication or surgical inter"ention as it would lead to more se"ere e7ocrine deiciency.
&u( 2. 0hile the possi/le presence o pancreatic cancer is an indication or surgery) there is no indication or
prophylactic surgery to decrease the ris! o cancer.
'-. Clinical Therapeutics/Cardiology
A $B year-old male presents with complaint o su/sternal chest pain or 1$ hours. The patient states that the
pain radiates to his shoulders and is relie"ed with sitting orward. The patient admits to recent upper respiratory
symptoms. ;n e7amination "ital signs are 3P 1$6/6-) HR -6) RR $,) temp 1,,.. degrees E. There is no ?42
noted. Heart e7am re"eals regular rate and rhythm with no 8. or 8'. There is a riction ru/ noted. Dungs are
clear to auscultation. CK< shows diuse 8T segment ele"ation. 0hat is the treatment o choice in this patient1
A. Pericardiocentesis
3. +itroglycerin
C. Percutaneous coronary inter"ention
2. *ndomethacin &*ndocin(
&u( A. Pericardiocentesis is the treatment o choice in a patient with a pericardial eusion and cardiac
tamponade) there is no e"idence o either o these in this patient.
&u( 3. +itroglycerin is indicated in the treatment o chest pain related to angina.
&u( C. Percutaneous coronory inter"ention is the treatment o choice in a patient with an acute myocardial
inarction.
&c( 2. *ndomethacin) a nonsteroidal anti-inlammatory medication) is the treatment o choice in a patient with
acute pericarditis.
'B. Clinical *nter"ention/2ermatology
As a general rule) sutures in the ace should /e remo"ed in
A. . days.
3. % days.
C. > days.
2. 1, days.
&c( 3. 8utures o the ace should /e remo"ed in % days in order to allow or ade@uate healing and to limit the
amount o scarring.
&u( A) C) 2 8ee 3 or e7planation.
%,. Health 6aintenance/;/stetrics/<ynecology
Patient education or a $. year-old using oral contracepti"es should include which o the ollowing1
A. Riampin may decrease the eecti"eness o the oral contracepti"es.
3. Acetaminophen may decrease the eecti"eness o the oral contracepti"es.
C. ;ral contracepti"es may pro"ide some protection rom coronary artery disease.
2. Changing to the GminipillG &progestin only( will inhi/it o"ulation more consistently than com/ination oral
contracepti"es.
&c( A. Riampin may interere with the eicacy o the oral contracepti"es.
&u( 3. Acetaminophen le"els or eects may /e decreased /y oral contracepti"es.
&u( C. Coronary artery disease is a contraindication to the use o oral contracepti"es.
&u( 2. Progestin only are less eecti"e at inhi/iting o"ulation than the com/ination oral contracepti"e.
%1. History : Physical/Arology/Renal
0hen perorming a rectal e7amination) prostatic massage is contraindicated in
A. acute /acterial prostatitis.
3. chronic /acterial prostatitis.
C. non/acterial prostatitis.
2. prostatodynia.
&c( A. 4igorous manipulation o the prostate during rectal e7amination may result in septicemia. This is
contraindicated in the presence o e"er) irritati"e "oiding symptoms) and perineal/sacral pain.
&u( 3. Prostate massage can /e perormed in the a/sence o e"er. C7pressed prostatic secretions are cultured to
help identiy the organism.
&u( C. +on/acterial prostatitis is similar to chronic /acterial prostatitis) /ut no /acteria are cultured) and the
cause may /e un!nown.
&u( 2. Prostatodynia is a noninlammatory disorder in"ol"ing "oiding dysunction and pel"ic loor musculature
dysunction. There is no /acterial in"ol"ement.
%$. Clinical *nter"ention/Arology/Renal K
A patient with prostate cancer has a nonpalpa/le) ocal lesion) and the patient is reluctant to ha"e surgery at this
time. 0hich o the ollowing would /est monitor disease progression1
A. Periodic rectal e7ams
3. Transrectal ultrasonography
C. 6easurements o serum acid phosphatase
2. 6easurements o prostate-speciic antigen
&u( A. 6any prostate carcinomas are contained within the gland) ma!ing it diicult to assess progression with a
digital e7amination alone.
&u( 3. Altrasonography is used largely or staging disease) not monitoring disease progression.
&u( C. 8erum acid phosphatase is more predicti"e o metastatic disease than P8A measurement) /ut its use has
largely /een replaced /y P8A.
&c( 2. P8A measurement correlates well with "olume and stage o disease and is the recommended e7amination
or monitoring disease progression.
%.. 8cientiic Concepts/;/stetrics/<ynecology
* a woman has a normal $--day menstrual cycle what tissue and hormonal phase occurs during the last 1'
days1
A. Prolierati"e ollicular phase under the inluence o estrogen.
3. 8ecretory luteal phase under the inluence o estrogen and progesterone.
C. Prolierati"e ollicular phase under the inluence o estrogen and progesterone.
2. 8ecretory luteal phase under the inluence o estrogen.
&c( 3. The endometrial changes seen in the latter hal o the cycle are under the inluence o /oth estrogen and
progesterone rom the corpus luteum. 2uring this phase) the endometrium /ecomes more "asculari9ed and
slightly edematous.
&u( A) C) 2 8ee 3 or e7planation.
%'. 2iagnosis/Cardiology
A $' year-old male presents or routine physical e7amination. ;n physical e7amination) you ind that the
patient5s upper e7tremity /lood pressure is higher than the /lood pressure in the lower e7tremity. Heart e7am
re"eals a late systolic murmur heard /est posteriorly. 0hat is the most li!ely diagnosis in this patient1
A. Hypertrophic o/structi"e cardiomyopathy
3. Patent oramen o"ale
C. Coarctation o the aorta
2. Patent ductus arteriosus
&u( A. Patients with hypertrophic o/structi"e cardiomyopathy do not present with hypertension or wea! emoral
pulses.
&u( 3. The murmur associated with patent oramen o"ale is a systolic e=ection murmur heard in the second and
third intercostal spaces and patients do not present with hypertension.
&c( C. Coarctation o the aorta commonly presents with higher systolic pressures in the upper e7tremities than
the lower e7tremities and a/sent or wea! emoral pulses.
&u( 2. Patent ductus arteriosus is rare in adults and patients are noted to ha"e a continuous rough) machinery
murmur.
%%. 2iagnosis/2ermatology
A mother /rings a . month-old inant to the oice /ecause she is concerned a/out a red) "ascular) nodular
growth on the child5s /ac!. *t appears to /e enlarging slightly and the "essels are slightly dilated. *t seems to
cause the child no discomort. The most li!ely diagnosis is
A. a hemangioma.
3. a pigmented ne"us.
C. a salmon patch &stor! /ite(.
2. a malignant melanoma.
&c( A. A hemangioma is a /right red to deep purple "ascular nodule or pla@ue that oten de"elops at /irth) may
enlarge) and may regress and disappear with aging.
&u( 3. A pigmented ne"us is not a "ascular lesion.
&u( C. A salmon patch is a light red macule o"er the nape o the nec! or the gla/ella.
&u( 2. A malignant melanoma presents as a hyperpigmented) asymmetric lesion with irregular /orders and is
typically seen in the adult population.
%6. Clinical Therapeutics/Cndocrinology
A '% year-old male with Type 1 dia/etes presents with the ollowing lipid panel# Total cholesterol .$1 mg/dDF
Triglycerides $$% mg/dDF H2D ., mg/dDF D2D 1%% mg/dD. The treatment o choice or this patient is
A. +icotinic acid &+iacin(.
3. Cholestyramine &Huestran(.
C. <emi/ro9il &Dopid(.
2. 8im"astatin &Jocor(.
&u( A. +iacin is not indicated in patients with dia/etes as it may worsen /lood sugar control.
&u( 3. Cholestyramine is not indicated as it may worsen the triglyceride le"el in this patient.
&u( C. <emi/ro9il should /e a"oided as it may worsen the D2D le"el.
&c( 2. 8im"astatin is the drug o choice as it will decreases triglyceride le"el) decrease D2D) and increase H2D.
%>. Clinical Therapeutics/<astrointestinal/+utritional
A '% year-old male presents with a/dominal pain and one episode o mild hematemesis) which happened days
ago. ;n physical e7amination) "ital signs are sta/le and he is in no acute distress. Hemoglo/in and hematocrit
are unremar!a/leF endoscopy re"eals non-/leeding small supericial ulceration o the duodenal /ul/. Rapid
urease test is positi"e. 0hich o the ollowing is the most appropriate treatment at this time1
A. 8chedule or a selecti"e "agotomy and antrectomy
3. 8tart an antacid along with omepra9ole &Prilosec(
C. 8chedule electi"e ulcer e7cision and start sucralate &Caraate(
2. 8tart omepra9ole &Prilosec( and anti/iotic therapy against H. pylori
&u( A. 6edical therapy should /e initiated prior to any consideration o surgery) which is rarely perormed
secondary to satisactory ulcer healing with medical therapy.
&u( 3. 0hile proton pump inhi/itors) such as omepra9ole) ha"e e7cellent results in healing duodenal ulcers) this
regimen will not treat the H. pylori inection that is documented /y the positi"e rapid urease test.
&u( C. 0hile sucralate can /e utili9ed as a cytoprotectant agent in treatment o acti"e ulcer disease or in
maintenance o healed ulcers) surgery to remo"e the ulcer is not warranted as initial therapy.
&c( 2. Treatment goals o H. pylori associated ulcers include eradicating the inection with appropriate
anti/iotics as well as use o a proton pump inhi/itor) such as omepra9ole) to promote ulcer healing.
%-. 2iagnostic 8tudies/*nectious 2iseases
A 1B year-old emale presents with a sore throat or nearly two wee!s. 8he complains o atigue and a low-
grade e"er. ;n physical e7amination) there is cer"ical) a7illary) and inguinal lymphadenopathy) and mild
splenomegaly. ;n re"iew o the /lood smear) which o the ollowing would /e e7pected1
A. Atypical lymphocytes
3. Hypersegmented neutrophils
C. Hypochromic red /lood cells
2. 8chistocytes
&c( A. The hallmar! o inectious mononucleosis is the presence o lymphocytosis with atypical large
lymphocytes seen in the /lood smear. These are larger than normal mature lymphocytes) stain more dar!ly) and
re@uently show "acuolated) oamy cytoplasm) and dar! chromatin in the nucleus.
&u( 3. Hypersegmented neutrophils are seen in "itamin 31$ deiciency.
&u( C. Anemia) i seen in mononucleosis) is normocytic and normochromic.
&u( 2. 8chistocytes are noted in hemolytic anemias.
%B. Health 6aintenance/Arology/Renal
0hich o the ollowing increases the ris! o de"eloping testicular cancer1
A. Dow socioeconomic status
3. History o cryptorchidism
C. 6ultiple episodes o epididymitis
2. 3eing o Arican-American ethnicity
&u( A. High socioeconomic status) not low) is a ris! actor.
&c( 3. The ma=or predisposing ris! actor is cryptorchidism unrepaired until ater age two.
&u( C. 6ultiple episodes o epididymitis are unrelated to the de"elopment o testicular cancer.
&u( 2. The incidence o testicular cancer is much lower in Arican-American men than in Caucasian men.
6,. 2iagnostic 8tudies/Psychiatry/3eha"ioral 6edicine
A 1> year-old patient presents to the emergency department with agitation and hallucinations) and has one
sei9ure. He admits to using Gsome drugsG /ut does not !now what they were. ;n physical e7amination)
temperature is 1,. degrees E) 3P 1',/B,) pulse 1$,) respirations $,. Remainder o the e7amination is
unremar!a/le. 0hich o the ollowing diagnostic studies will /e o most help in managing this patient1
A. 2rug screen
3. Arine dipstic!
C. Complete /lood count
2. 8erum creatinine !inase
&a( A. Although a drug screen may identiy speciic drugs) the results will not alter the care o this patient.
&u( 3. Arine dipstic! is not sensiti"e or myoglo/inuria.
&u( C. This patient is at ris! or myoglo/inuria) and a complete /lood count will not alter the treatment.
&c( 2. 8erum creatinine !inase is the most sensiti"e test to detect rha/domyolysis) a serious complication o
sei9ures and hyperthermia related to drug a/use.
61. Clinical Therapeutics/;rthopedics/Rheumatology
0hich o the ollowing medications is the treatment o choice or patients with chronic gout to pre"ent
recurrence o symptoms during its @uiescent phase1
A. Pro/enecid &3enemid(
3. Allopurinol &Jyloprim(
C. Colchicine
2. *ndomethacin &*ndocin(
&u( A. Pro/enecid is a uricosuric medication that helps to increase the e7cretion o uric acid /ut it does not
pre"ent the ormation o uric acid ma!ing it less /eneicial in chronic gout therapy. *t is also not eecti"e in
patients nwith chronic renal disease.
&c( 3. Allopurinol is the /est drug to lower serum urate in o"erproducers) stone ormers) and patients with
ad"anced renal ailure. *t is a 7anthine o7idase inhi/itor that is used to pre"ent the ormation o uric acid.
&u( C. Colchicine treatment is recommended only in patients who ha"e tophaceous deposits in the s!in and is
used in the acute rather than the chronic setting o gout.
&u( 2. *ndomethacin is used in the acute management o gout /ut is not eecti"e in decreasing monosodium
urate deposition in the =oints.
6$. 2iagnosis/Cndocrinology
A 6' year-old male presents complaining o new onset o atigue) weight gain) constipation) erectile
dysunction) and loss o /ody hair. Da/oratory in"estigation demonstrates# T8H less than ,.% microunits/mD
&normal range ,.%-%., microA/mD(F Thyro7ine &T'( $ mcg/dD &normal range %-1$ mcg/dD(F Prolactin 1,
nanograms/ml
A. Primary hypothyroidism
3. C7cessi"e dosing o le"othyro7ine &8ynthroid(
C. Hypopituitarism
2. 8u/acute thyroiditis
&u( A. Primary hypothyroidism is usually associated with an ele"ated T8H.
&u( 3. C7cessi"e replacement o le"othyro7ine would result in symptoms o hyperthyroidism) not
hypopituitarism.
&c( C. The low trophic and target hormone le"els com/ined with symptoms o hypogonadism indicate this
patient has hypopituitarism.
&u( 2. The T' le"el would /e ele"ated in su/acute thyroiditis due to e7cessi"e release o thyroid hormone.
6.. 2iagnosis/Pulmonology
A 1% year-old male was seen last wee! with complaints o sore throat) headache) and mild cough. A diagnosis
o AR* was made and supporti"e treatment was initiated. He returns today with complaints o worsening cough
and increasing atigue. At this time) chest 7-ray re"eals /ilateral hilar iniltrates. A 03C count is normal and a
cold hemagglutinin titer is ele"ated. The most li!ely diagnosis is
A. tu/erculosis.
3. mycoplasma pneumonia.
C. pneumococcal pneumonia.
2. staphylococcal pneumonia.
C7planations
&u( A. 6ost children with pulmonary tu/erculosis are asymptomatic with ew physical e7amination indings.
The results o the diagnostic studies do not support tu/erculosis as the most li!ely diagnosis.
&c( 3. The insidious onset o symptoms) the interstitial iniltrate on chest 7-ray) and ele"ated cold hemagglutinin
titer ma!e this diagnosis the most li!ely.
&u( C. The clinical presentation o /acterial pneumonias in children is "aria/le) /ut usually in"ol"es e"er o
acute onset. The 03C count is also usually ele"ated) ma!ing this a less li!ely diagnosis.
&u( 2. 8ee C or e7planation.
6'. History : Physical/;/stetrics/<ynecology
0hich o the ollowing clinical maniestations is common in candidal "ul"o"aginitis1
A. C7treme "ul"ar irritation
3. Eirm) painless ulcer
C. Tender lymphadenopathy
2. Purulent discharge
C7planations
&c( A. Candida inection presents with pruritus) "ul"o"aginal erythema) and white) cheese-li!e &curd( discharge
that may /e malodorous.
&u( 3. A irm painless ulcer is seen in syphilis.
&u( C. Tender lymphadenopathy is associated with /acterial inections and is not a eature o candidal
"ul"o"aginitis.
&u( 2. Purulent discharge is noted in gonorrhea.
6%. 2iagnosis/Cardiology
A 6. year-old emale presents with a complaint o chest pressure or one hour) noticed upon awa!ening. 8he
admits to associated nausea) "omiting) and shortness o /reath. 1$ lead CK< re"eals 8T segment ele"ation in
leads **) ***) and A4E. 0hich o the ollowing is the most li!ely diagnosis1
A. Aortic dissection
3. *nerior wall myocardial inarction
C. Acute pericarditis
2. Pulmonary em/olus
&u( A. A patient with aortic dissection will complain o tearing) ripping pain. CK< is oten normal) /ut may
re"eal let "entricular strain pattern.
&c( 3. 6yocardial inarction oten presents with chest pressure and associated nausea and "omiting. 8T segment
ele"ation in leads **) ***) and A4E are classic indings seen in acute inerior wall myocardial inarction.
&u( C. Acute pericarditis presents with atypical chest pain and diuse 8T segment ele"ation.
&u( 2. Pulmonary em/olism oten presents with either no CK< changes or sinus tachycardia. Classically
descri/ed) rarely seen indings include a large 8 wa"e in lead *) a H wa"e with T wa"e in"ersion in lead ***) 8T
segment depression in lead **) T wa"e in"ersion in leads 41-4' and a transient right /undle /ranch /loc!.
66. 2iagnostic 8tudies/Hematology
An 1- year-old woman presents to the clinic complaining o atigue. 8he reports a past history o lielong
re@uent nose/leeds and /leeding gums. 8he also has menorrhagia. Her mother and maternal grandather ha"e a
similar /leeding history. *nitial la/ results are as ollows# 03C B)%,,/mm.) Hg/ 1,.B g/dD) HCT ..I) 6C4
6B D) 6CHC $6 pg and platelets $-'),,,/mm.. 0hich o the ollowing tests should /e ordered to e"aluate this
patient5s diagnosis1
A. Hemoglo/in electrophoresis
3. 3leeding time and platelet aggregometry
C. 3one marrow aspiration
2. PT and aPTT
&u( A. Hemoglo/in electrophoresis would /e utili9ed to e"aluate microcytic) hypochromic anemias.
&c( 3. The patient5s presentation is consistent with a congential @ualitati"e platelet disorder) most li!ely "on
0ille/rand5s 2isease) necessitating a /leeding time and e"aluation o platelet unction.
&u( C. 3one marrow aspiration is not utili9ed in the e"aluation o @ualitati"e platelet disorders.
&u( 2. A PT and aPTT would /e utili9ed to e"aluate or /leeding consistent with a/normalities with the
coagulation cascade.
6>. Clinical Therapeutics/C+T/;phthalmology
A .% year-old patient has recurrent seasonal rhinitis and a history o mild asthma. 0hich o the ollowing
should /e included or irst-line management1
A. *mmunotherapy
3. 2econgestants
C. Corticosteroid inhalers
2. Cromolyn sodium &*ntal(
&u( A. *mmunotherapy &desensiti9ation( is indicated as a last resort in patients who ail to either respond to
pharmaceutical management or ace prolonged e7posure to !nown allergens.
&u( 3. 2econgestants ha"e a limited role in helping to decrease edema) and are generally ineecti"e in relie"ing
allergic symptoms.
&c( C. Regular use o corticosteroid nasal spray and oral inhalers prior to the allergy season is among the /est
means o pre"enting allergies.
&u( 2. Cromolyn sodium has /een ound to /e moderately eecti"e or some patients with allergic symptoms)
/ut it is not usually irst-line management.
6-. 2iagnosis/;/stetrics/<ynecology
A .% year-old emale presents with multiple ulcerati"e lesions on her la/ia and perineum. A T9anc! preparation
o one o the lesions re"eals multinucleated giant cells. 0hich o the ollowing is the most li!ely diagnosis1
A. Herpes 8imple7 4irus &H84(
3. 6olluscum Contagiosum 4irus &6C4(
C. Human Papilloma 4irus &HP4(
2. 8yphilis
&c( A. The presentation seen on the T9anc! preparation is characteristic o H84.
&u( 3) C) 2. 8ee A or e7planation.
6B. 8cientiic Concepts/C+T/;phthalmology
8mall grayish "esicles and punched-out ulcers in the posterior pharyn7 in a child with pharyngitis is
representati"e o which organism1
A. Cpstein-3arr "irus
3. <roup C 8treptococcus
C. Co7sac!ie"irus
2. <onorrhea
&u( A. Cpstein-3arr "irus presents with enlarged tonsils with e7udates and petechiae o the palate.
&u( 3. <roup C 8treptococcus presents with a red pharyn7 and enlarged tonsils with a yellow) /lood tinged
e7udates
&c( C. Co7sac!ie"irus presents with small grayish "esicles and punched-out ulcers in the posterior pharyn7.
&u( 2. +eisseria gonorrhea o the pharyn7 may /e asymptomatic
>,. Health 6aintenance/+eurology
A %. year-old emale has a diagnosis o migraine headaches. 8he had /een using sumatriptan &*mitre7( to a/ort
her headaches) /ut she is now ha"ing 1 or $ headaches per wee!. The most appropriate pre"enti"e therapy is
A. 9olmitriptan &Jomig(.
3. prometha9ine &Phenergan(.
C. propranolol &*nderal(.
2. luo7etine &Pro9ac(.
&u( A. Jolmitriptan) another %-HT receptor agonist) is used acutely to a/ort migraine headaches.
&u( 3. Prometha9ine is an antiemetic that may /e used to alle"iate nausea and "omiting rom an acute migraine.
&c( C. Propanolol is useul in pre"enting migraine headaches and may /e maintained indeinitely.
&u( 2. Eluo7etine is an 88R* that is used in the treatment o an7iety and o/sessi"e-compulsi"e disorders.
>1. History : Physical/+eurology Topic# . Author#
0hich o the ollowing primiti"e rele7es should /egin to disappear at a/out $ months o age in a normal
inant1
A. 6oro
3. <rasp
C. Tonic nec!
2. Parachute
&u( A. The 6oro rele7 starts to disappear at a/out %-6 months o age.
&c( 3. The grasp rele7 starts to disappear at a/out $-. months o age.
&u( C. The tonic nec! rele7 starts to disappear at a/out 6-> months o age.
&u( 2. The parachute rele7 remains throughout lie.
>$. History : Physical/Cardiology
A 1$ month-old child with tetralogy o Eallot is most li!ely to ha"e which o the ollowing clinical eatures1
A. Chest pain
3. Cyanosis
C. Con"ulsions
2. Palpitations
&u( A. Chest pain is not a eature o tetralogy o Eallot.
&c( 3. Cyanosis is "ery common in tetralogy o Eallot.
&u( C. Con"ulsions are occasionally seen as part o se"ere hypo7ic spells in inancy rather than a eature o
tetralogy o Eallot.
&u( 2. Palpitations are uncommon in tetralogy o Eallot.
>.. Clinical Therapeutics/;rthopedics/Rheumatology
*ntraarticular in=ection o hyaluronic acid has /een appro"ed or treatment o patients with which o the
ollowing conditions1
A. Rheumatoid arthritis o the !nee
3. ;steoarthritis o the !nee
C. ;lecranon /ursitis
2. <outy arthritis
&u( A. 8ee 3 or e7planation.
&c( 3. *ntraarticular in=ection o hyaluronic acid has /een appro"ed recently or treatment o patients with
osteoarthritis o the !nee that ha"e ailed other therapies. Although the onset o action o this medication is
slower than in=ected glucocorticoids) it has a sustained length o acti"ity outlasting the in=ected glucocorticoids.
&u( C. Treatment o olecranon /ursitis may include incision and drainage /ut not hyaluronic acid in=ections.
&u( 2. <out is treated with anti-inlammatory medications.
>'. Clinical Therapeutics/Psychiatry/3eha"ioral 6edicine
0hich o the ollowing drugs is irst-line therapy or schi9ophrenia1
A. Chlorproma9ine &Thora9ine(
3. Clo9apine &Clo9aril(
C. Haloperidol &Haldol(
2. ;lan9apine &Jypre7a(
C7planations
&u( A. The older) traditional antipsychotic agents) such as haloperidol and chlorproma9ine ha"e higher ris! o
side eects) including acute motor system side eects a long-term ris! o tardi"e dys!inesias) and should not /e
considered as irst-line drugs.
&u( 3. Clo9apine should not /e considered a irst-line therapy /ecause o its hematopoietic and hepatic side
eects.
&u( C. 8ee A or e7planation.
&c( 2. *nitial pharmacologic therapy o schi9ophrenia should /egin with one o the newer) GatypicalG
antipsychotic drugs) such as olan9apine) risperidone) @uetiapine) 9iprasidone) and clo9apine /ecause their side
eect proile is signiicantly /etter than the older drugs) and they may /e more eecti"e or negati"e psychotic
symptoms.
>%. 2iagnosis/;rthopedics/Rheumatology
A $% year-old presents with pain in the pro7imal ulna ater alling directly on the orearm. L-ray shows racture
o the pro7imal 1/.rd o the ulna. There is an associated anterior radial head dislocation. 0hat is the proper
name or this condition1
A. <alea99i racture
3. 6onteggia racture
C. Colles5 racture
2. 8mith racture
&u( A. A <alea99i racture is a racture along the length o the radius with in=ury to the distal radioulnar =oint.
&c( 3. A 6onteggia racture is a racture o the pro7imal ulna with anterior dislocation o the radial head.
&u( C. A Colles5 racture is a racture o the distal radius with dorsal displacement o the radial head.
&u( 2. A 8mith racture is a racture o the distal radius with "entral displacement o the radial head.
>6. 2iagnosis/<astrointestinal/+utritional
A $, year-old male presents with a mass in the groin. ;n e7amination with the patient standing) a mass is noted
that e7tends into the scrotum. The patient denies any trauma. The most li!ely diagnosis is
A. an indirect inguinal hernia.
3. a direct inguinal hernia.
C. an o/turator hernia.
2. a emoral hernia.
&c( A. An indirect inguinal hernia is caused /y a patent processus "aginalis and the hernial contents may /e elt
in the ipsilateral scrotum.
&u( 3. A direct inguinal hernia is symmetrical) round and disappears easily with the patient lying down. *t is the
result o a wea!ness in the inguinal .e7ternal ring. Hernial contents may radiate anteriorly rather than into the
scrotum.
&u( C. ;/turator hernia5s are more commonly seen in elderly women and are rarely palpa/le in the groin.
&u( 2. Eemoral hernias are rare in males and do not typically reduce with lying down.
>>. Health 6aintenance/C+T/;phthalmology
A patient with type $ dia/etes mellitus presents or a yearly eye e7am. ;phthalmoscopic e7am re"eals
neo"asculari9ation. 0hich o the ollowing is the most li!ely complication related to this inding1
A. <laucoma
3. Cataracts
C. 4itreous hemorrhage
2. ;ptic neuritis
&u( A. <laucoma occurs in a/out 6I o dia/etics. +eo"asculari9ation o the iris can cause closed angle
glaucoma.
&u( 3. Cataracts can occur secondary to dia/etes) /ut are not caused /y prolierati"e retinopathy.
&c( C. Prolierati"e retinopathy) as e"idenced /y neo"asculari9ation) is associated with an increased ris! o
"itreous hemorrhage.
&u( 2. ;ptic neuritis is strongly associated with demyelinating disease) li!e multiple sclerosis.
>-. Clinical Therapeutics/Cndocrinology
0hich o the ollowing oral hypoglycemic agents when used as monotherapy is most li!ely to cause
hypoglycemia1
A. <lipi9ide &<lucotrol(
3. 6etormin &<lucophage(
C. Pioglita9one &Actos(
2. Acar/ose &Precose(
&c( A. 8ulonylureas increase insulin le"els and predispose patients to hypoglycemia.
&u( 3. 3iguanides cause gastritis and lactic acidosis
&u( C. T2Js
&u( 2. Acar/ose is an alpha glucosidase inhi/itor that delays the a/sorption o car/ohydrates in the diet. *t does
not cause hypoglycemia since it does not dri"e insulin into the cells.
>B. 2iagnosis/;rthopedics/Rheumatology
A >% year-old emale presents with medial !nee pain that worsens with stair clim/ing. Physical e7amination
re"eals swelling and point tenderness inerior and medial to the patella and tenderness o"erlying the medial
ti/ial plateau. 0hich o the ollowing is the most li!ely diagnosis1
A. Pes anserine /ursitis
3. Prepatellar /ursitis
C. *nrapatellar /ursitis
2. Trochanteric /ursitis
&c( A. The pes anserine /ursa underlies the semimem/ranosus tendon and may /ecome inlamed or painul
owing to trauma) o"eruse) or inlammation. *t is a common cause o !nee pain and it is oten misdiagnosed in
adults.
&u( 3. Prepatellar /ursitis causes swelling in the prepatellar area and is worse with !neeling. The prepatellar
/ursa is nsupericial and is located o"er the inerior portion o the patella.
&u( C. The inrapatellar /ursa is deeper and lies /eneath the patellar ligament /eore its insertion on the ti/ial
ntu/ercle. *t has a midline location rather than the medial surace as descri/ed in the @uestion.
&u( 2. Trochanteric /ursitis causes hip pain rather than !nee pain.
-,. 2iagnosis/Cardiology
A $. year-old male presents with syncope. ;n physical e7amination you note a medium-pitched) mid-systolic
murmur that decreases with s@uatting and increases with straining. 0hich o the ollowing is the most li!ely
diagnosis1
A. Hypertrophic cardiomyopathy
3. Aortic stenosis
C. 6itral regurgitation
2. Pulmonic stenosis
&c( A. Hypertrophic cardiomyopathy is characteri9ed /y a medium- pitched) mid-systolic murmur that
decreases with s@uatting and increases with straining.
&u( 3. 8training decreases the intensity o the murmur associated with aortic stenosis and s@uatting increases the
intensity.
&u( C. 6itral regurgitation is characteri9ed /y a /lowing systolic murmur that radiates to the a7illa) it is not
oten associated with syncope.
&u( 2. Pulmonic stenosis is a harsh systolic murmur with a widely split 8$) and no change with maneu"ers.
-1. Clinical Therapeutics/<astrointestinal/+utritional
0hich o the ollowing can /e a "ery serious conse@uence o using antidiarrheals in a patient with inlammatory
/owel disease1
A. Dymphoma
3. To7ic megacolon
C. 3one marrow suppression
2. 2elayed serum sic!ness-li!e reaction
&u( A. There is suggestion that lymphoma may result rom the use o inli7ima/) an anti-T+E agent) utili9ed in
reractory cases o Crohn5s diseaseF howe"er a clear lin! /etween the de"elopment o lymphoma and the
use o inli7ima/ has not /een esta/lished.
&c( 3. Antidiarrheals may cause the de"elopment o to7ic megacolon when used /y patients with acti"e se"ere
inlammatory /owel disease.
&u( C. 3one marrow suppression is a side eect o methotre7ate) which is /eing increasingly used in the
treatment o Crohn5s disease.
&u( 2. 2elayed serum sic!ness-li!e reaction is another potential side eect o inli7ima/.
-$. 2iagnosis/;rthopedics/Rheumatology
A $- year-old emale) who has e7perienced occasional painul migratory arthralgias) complains now o a tender)
swollen) and hot let an!le. The =oint was aspirated and the syno"ial luid showed %%),,, 03Cs) >%I
polymorphonuclear lymphocytes) low glucose le"el) and no crystals. 0hich o the ollowing would /e the most
li!ely diagnosis1
A. Rheumatoid arthritis
3. 8eptic arthritis
C. <outy arthritis
2. ;steoarthritis
&u( A. Rheumatoid arthritis usually in"ol"es more than one =oint) primarily aecting metacarpophalangeal and
wrist =oints initially. 8yno"ial luid analysis would show moderate leu!ocytosis &M %,),,,( with higher glucose
le"els.
&c( 3. 8eptic arthritis presents with a large num/er o 03Cs) predominantly polymorphonuclear) and with
glucose le"els much lower than serum le"els.
&u( C. Although gout presents as monarticular process) it usually presents acutely without pre"ious migratory
arthralgias. Crystals are usually ound in syno"ial analysis.
&u( 2. ;steoarthritis eusions may /e normal or show mild ele"ations o 03Cs in the luid analysis) /ut it is
rarely monarticular.
-.. Clinical Therapeutics/Arology/Renal
0hen the diagnosis o gonococcal urethritis is conirmed) which o the ollowing is the treatment o choice1
A. Cetria7one &Rocephin(
3. Amo7icillin &Amo7il(
C. Penicillin < /en9athine &3icillin DA(
2. 2o7ycycline &4i/ramycin(
C7planations
&c( A. Cetria7one is recommended therapy or gonococcal urethritis.
&u( 3. Cecti"e single-dose regimens or uncomplicated gonococcal urethritis include cei7ime or one o the
luoro@uinolones) /ut +;T amo7icillin due to penicillin-resistant strains o gonorrhea.
&u( C. Penicillin < /en9athine is the treatment o choice or primary syphilis.
&u( 2. 2o7ycycline is the treatment o choice or Chlamydia urethritis) not gonococcal urethritis.
-'. History : Physical/2ermatology
Crythema nodosum is characteri9ed /y
A. su/cutaneous red tender nodules.
3. /rown pigmentation on the lower e7tremities.
C. tender lymph nodes in the groin.
2. scaling red macules.
&c( A. Crythema nodosum produces erythematous red tender nodules) especially on the shins.
&u( 3. 3rown pigmentation on the lower e7tremities is a eature o chronic "enous insuiciency.
&u( C. Dympho"enereum granuloma is li!ely to produce tender lymph nodes in the groin as can acute inection
in the lower e7tremities.
&u( 2. 8caling red macules are a eature o tinea corporis.
-%. Clinical Therapeutics/C+T/;phthalmology
Ase o systemic corticosteroids can cause which o the ollowing ad"erse eects in the eye1
A. Cortical /lindness
3. ;ptic atrophy
C. <laucoma
2. Papilledema
&u( A. Cortical /lindness is a rare ad"erse eect when prescri/ing salicylates.
&u( 3. ;ptic atrophy can occur as an ad"erse eect with lead compounds) ame/icides) and 6A; inhi/itors.
&c( C. <laucoma can /e caused /y the long-term use o steroids.
&u( 2. Papilledema can /e a side eect to many systemic medications.
-6. Health 6aintenance/;/stetrics/<ynecology
*t is determined that a woman has a none7istent ru/ella titer le"el during her irst trimester o pregnancy. 0hen
should she recei"e the ru/ella "accine1
A. 2uring the irst trimester o pregnancy
3. 2uring the second trimester o pregnancy
C. 2uring the third trimester o pregnancy
2. Ater deli"ery o the inant
&h( A) 3) C. 8ee 2 or e7planation.
&c( 2. The patient should not recei"e the ru/ella "accine during the course o her pregnancy as the possi/ility o
transmission o the ru/ella "irus does e7ist. 2uring the time that the patient is without protecti"e titer she should
a"oid anyone with acti"e ru/ella inection. The proper time to recei"e the "accine is ater deli"ery o the inant.
->. Health 6aintenance/Cardiology
A patient with which o the ollowing is at highest ris! or coronary artery disease1
A. Congenital heart disease
3. Polycystic o"ary syndrome
C. Acute renal ailure
2. 2ia/etes mellitus
&u( A. Congenital heart disease is not an esta/lished ris! actor or coronary artery disease.
&u( 3. 0hile patients with polycystic o"ary syndrome ha"e hyperinsulimemia) they do not ha"e the same poor
prognosis or coronary artery disease as patients with dia/etes mellitus.
&u( C. Patients with acute renal ailure are not at ris! or coronary artery disease) although patients with dia/etes
and chronic renal disease do ha"e this ris!.
&c( 2. Patients with dia/etes mellitus are in the same ris! category or coronary artery disease as those patients
with esta/lished atherosclerotic disease.
--. 2iagnostic 8tudies/;rthopedics/Rheumatology
A '' year-old emale presents with ongoing arthralgias and myalgias with intermittent lares o arthritis. 8he is
ound to ha"e a malar rash that worsens with sun e7posure. 8he is !nown to ha"e progressi"e renal damage and
has recurrent inections that are slow to respond to therapy. 8he ta!es i/uproen &6otrin( as needed or her =oint
pain and ta!es no other medication. 0hich o the ollowing tests would /e the initial test recommended to
screen or this diagnosis1
A. Rheumatoid actor
3. Antihistone anti/odies
C. Anti-8mith &Anti-8m( anti/odies
2. Anti-nuclear anti/odies &A+A(
&u( A. Rheumatoid actor is most commonly perormed in the assessment o rheumatoid arthritis and not
suspected systemic lupus erythematosus.
&u( 3. Antihistone anti/odies are seen in drug-induced 8DC rather than in spontaneously occurring 8DC.
&u( C. Anti-8mith anti/odies and anti/odies to the dou/le stranded 2+A are airly speciic to 8DC /ut their role
is or conirmation rather than screening or 8DC.
&c( 2. A+A5s are the /est screening test used in the e"aluation or 8DC.
-B. History : Physical/+eurology
Apon stro!ing o the lateral aspect o the sole rom the heel to the /all o the oot) the great toe dorsile7es and
the other toes an. This is a positi"e
A. Kernig5s sign.
3. 3rud9ins!i5s sign.
C. 3a/ins!i5s sign.
2. <ower5s sign.
&u( A. Kernig5s sign is positi"e when pain is noted on straightening the !nee ater le7ing /oth the hip and !nee.
&u( 3. 3rud9ins!i5s sign occurs with nec! le7ion resulting in resultant le7ion o the hips. *t is a sign o
meningeal irritation.
&c( C. A 3a/ins!i test is perormed /y stro!ing the lateral aspect o the sole rom the heel to the /all o the oot)
the great toe dorsile7es and the other toes an in a positi"e test.
&u( 2. A positi"e <ower5s sign is noted in certain types o muscular dystrophy and is descri/ed as children
rising to stand /y rolling o"er prone and pushing o the loor with arms while the legs remain e7tended.
B,. Health 6aintenance/Cndocrinology
0hich o the ollowing strategies promotes impro"ed car/ohydrate meta/olism and is recommended or all
Type $ dia/etic patients1
A. Dow-car/ohydrate) high protein diet
3. Routine aero/ic e7ercise
C. 6etormin &<lucophage(
2. Acupuncture
&u( A. There is no e"idence that a low-car/ohydrate) high protein diet is eecti"e in impro"ing car/ohydrate
meta/olism.
&c( 3. Routine e7ercise impro"es car/ohydrate meta/olism and insulin sensiti"ity.
&u( C. 6etormin is a /iguanide that primarily wor!s to decrease gluconeogenesis rather than aecting
car/ohydrate meta/olism.
&u( 2. There is no e"idence that acupuncture impro"es car/ohydrate meta/olism.
B1. Clinical Therapeutics/Cardiology
Acute re/ound hypertensi"e episodes ha"e /een reported to occur with the sudden withdrawal o
A. "erapamil &Calan(.
3. lisinopril &Prini"il(.
C. clonidine &Catapres(.
2. hydrochlorothia9ide &HCTJ(
&u( A. 4erapamil is a calcium channel /loc!er and there is no associated re/ound hypertension ater withdrawal.
&u( 3. Disinopril is an ACC inhi/itor) which is not associated with re/ound hypertension.
&c( C. Clonidine &Catapres( is a central alpha agonist and a/rupt withdrawal may produce a re/ound
hypertensi"e crisis.
&u( 2. Hydrochlorothia9ide is a thia9ide diuretic) which is not associated with re/ound hypertension.
B$. Clinical *nter"ention/;/stetrics/<ynecology
A >$ year-old emale presents with "ul"ular pruritus or the last nine months) which has progressi"ely worsened
o"er the last two months. 8he states that she went through menopause at age %' and has /een on estrogen and
progesterone therapy since that time. Physical e7amination re"eals red lesions with white pla@ues on the "ul"a.
0hat should the ne7t course o management include1
A. Reer to a gynecologist or /iopsy.
3. Reer to a dermatologist or antiungal therapy.
C. Treat with a topical steroid.
2. Treat with estrogen cream.
&c( A. 4ul"ular s@uamous cell hyperplasia causes thic!ening and hyper!eratosis o the "ul"a. The lesions are
red and moist and cause intense pruritus o"er time the area /ecomes thic!ened and a white pla@ue may de"elop.
3iopsy must /e done to e"aluate or intraepithelial neoplasm or in"asi"e tumor.
&u( 3)C)2. 8ee A or e7planation.
B.. 2iagnosis/;/stetrics/<ynecology
A ., year-old emale presents to the emergency department with a syncopal episode. 8he has a history o
irregular menstrual cycles and inertility. 8he has scanty) persistent "aginal /leeding and sharp pel"ic pain. A
let adne7al mass is palpated. The most li!ely diagnosis is
A. placenta a/ruptio.
3. ectopic pregnancy.
C. pel"ic inlammatory disease.
2. ruptured o"arian cyst.
&u( A. This is primarily a third trimester cause o "aginal /leeding.
&c( 3. *nertility increases the ris! o de"eloping ectopic pregnancy. The onset o "aginal /leeding) pel"ic pain)
and ormation o an adne7al mass ma!es this the most li!ely diagnosis.
&u( C. Pel"ic inlammatory disease typically presents with e"er) a/dominal pain) purulent "aginal discharge)
and cer"ical motion tenderness.
&u( 2. Acute pel"ic pain may occur secondary to /leeding rom the rupture o a hemorrhagic o"arian cyst) /ut
no adne7al mass would /e palpa/le on pel"ic e7amination.
B'. History : Physical/<astrointestinal/+utritional
A patient presents complaining o perium/ilical pain. 0hich o the ollowing anatomical sites is this inding
associated with1
A. 3ladder
3. 8tomach
C. Pancreas
2. 8mall /owel
&u( A. Pain rom the /ladder) uterus) or colon causes hypogastric pain.
&u( 3. Pain rom the stomach) duodenum) or pancreas causes epigastric pain.
&u( C. 8ee 3 or e7planation.
&c( 2. Pain rom the small intestine) appendi7) or pro7imal colon causes perium/ilical pain.
B%. 2iagnosis/;rthopedics/Rheumatology
A %. year-old patient presents with se"ere pain at the /ase o the thum/ and no other inger in"ol"ement. The
pain is worse with acti"ity and lasts a short period o time ollowing rest. There is no speciic history o trauma
to the thum/ /ut the patient admits wor!ing with her hands as a typist. 0hich o the ollowing is the most li!ely
diagnosis1
A. Rheumatoid arthritis
3. ;steoarthritis
C. Hemochromatosis
2. Pseudogout
&u( A. Rheumatoid arthritis typically in"ol"es the 6CP and P*P =oints o the digits or the second through ith
ingers. The thum/ is classically spared.
&c( 3. The /ase o the thum/ is typically in"ol"ed with osteoarthritis as are the 2*P =oints o the other ingers.
&u( C. Hemochromatosis classically in"ol"es the 6CP =oints o the second through ith ingers.
&u( 2. Pseudogout =oint in"ol"ement is typically the 6CP =oints o the second through ith ingers.
B6. 2iagnosis/Cardiology
A .- year-old emale with history o coarctation o the aorta repair at the age o two presents with e"ers or
our wee!s. The patient states that she has elt atigued and achy during this time. 6a7imum temperature has
/een 1,$.1 degrees E. 8he denies cough) congestion) or other associated symptoms. Physical e7amination
re"eals a pale tired appearing emale in no acute distress. Heart re"eals a new grade ***-*4/4* systolic e=ection
/order at the ape7) and a **/4* diastolic murmur at the right sternal /order. 0hat is the most li!ely diagnosis1
A. Acute myocardial inarction
3. 3acterial endocarditis
C. Acute pericarditis
2. Restricti"e cardiomyopathy
&u( A. Acute 6* presents with complaint o chest pain) 8;3) not with e"er and myalgias.
&c( 3. 3acterial endocarditis presents as e/rile illness lasting se"eral days to wee!s) commonly with
nonspeciic symptoms) echocardiogram oten re"eals "egetations on aected "al"es.
&u( C. Pericarditis does not present with systolic or diastolic murmur or "egetation) more commonly pericardial
riction ru/ would /e noted.
&u( 2. Restricti"e cardiomyopathy will show impaired diastolic illing on echocardiogram and is not associated
with e"er.
B>. Clinical Therapeutics/<astrointestinal/+utritional
A '% year-old male presents with a/dominal pain and one episode o mild hematemesis) which happened days
ago. ;n physical e7amination) "ital signs are sta/le and he is in no acute distress. Hemoglo/in and hematocrit
are unremar!a/leF endoscopy re"eals non-/leeding small supericial ulceration o the duodenal /ul/. Rapid
urease test is positi"e. 0hich o the ollowing is the most appropriate treatment at this time1
A. 8chedule or a selecti"e "agotomy and antrectomy
3. 8tart an antacid along with omepra9ole &Prilosec(
C. 8chedule electi"e ulcer e7cision and start sucralate &Caraate(
2. 8tart omepra9ole &Prilosec( and anti/iotic therapy against H. pylori
&u( A. 6edical therapy should /e initiated prior to any consideration o surgery) which is rarely perormed
secondary to satisactory ulcer healing with medical therapy.
&u( 3. 0hile proton pump inhi/itors) such as omepra9ole) ha"e e7cellent results in healing duodenal ulcers) this
regimen will not treat the H. pylori inection that is documented /y the positi"e rapid urease test.
&u( C. 0hile sucralate can /e utili9ed as a cytoprotectant agent in treatment o acti"e ulcer disease or in
maintenance o healed ulcers) surgery to remo"e the ulcer is not warranted as initial therapy.
&c( 2. Treatment goals o H. pylori associated ulcers include eradicating the inection with appropriate
anti/iotics as well as use o a proton pump inhi/itor) such as omepra9ole) to promote ulcer healing.
B-. History : Physical/Cndocrinology
0hich o the ollowing indings is usually associated with Addison5s disease1
A. 0eight gain
3. Hypertension
C. *ncreased pigmentation
2. High plasma cortisol le"els
&u( A. Patients with Addison5s disease tend to ha"e anore7ia and weight loss.
&u( 3. Patients with Addison5s disease tend to ha"e hypotension.
&c( C. Patients with Addison5s disease ha"e diuse tanning o"er none7posed and e7posed s!in due to increased
melanocytic actor that is released with adrenocorticotropic hormone.
&u( 2. Patients with Addison5s disease tend to ha"e low plasma cortisol le"els.
BB. Clinical *nter"ention/Pulmonology
A 6, year-old patient with C;P2 characteristic o emphysema presents with a cough and increased sputum
production. The ollowing inormation is noted# Temperature 1,,NE &.>.-NC(F Respiratory rate $,/minF Heart
rate -- /eats/minF pH >.''F Pa;$ >% mmHgF PaC;$ ', mmHgF ;$ saturation B$I. Physical e7amination is
remar!a/le or increased AP diameter) diminished /reath sounds without whee9es) rhonchi) or other signs o
respiratory distress. 0hich o the ollowing would /e an appropriate treatment or this patient1
A. 3road-spectrum anti/iotic
3. Admission to the hospital
C. ;7ygen at 6 D/min /y nasal cannula
2. 3rie course o oral theophylline
&c( A. 8putum production is e7tremely "aria/le rom patient to patient) /ut any increase in sputum with a
history o C;P2 reported /y a patient must /e regarded as potentially inectious and treated promptly.
&u( 3. Admission is only warranted i the patient5s respiratory status re@uires "entilatory assistance. This
patient5s /lood gases are unremar!a/le or a patient with C;P2 and the patient is not in respiratory distress.
&u( C. ;7ygen therapy should only /e used or se"ere hypo7emia and should only /e gi"en at a low
concentration) such as $ D/min. Higher dose o7ygen may stop the hypo7emic "entilatory dri"e.
&u( 2. ;ral theophylline is considered a secondary /ronchodilator. The use o a metered-dose inhaler would /e
a preera/le irst-line treatment i this method o treatment were chosen.
1,,. History : Physical/Hematology
0hich o the ollowing physical indings suggest pernicious anemia1
A. 8plenomegaly and hepatomegaly
3. Petechiae and ecchymosis
C. Doss o position and "i/ratory sensation
2. Cheilosis and !oilonychia
&u( A. 8plenomegaly and hepatomegaly are typically seen in hemolytic anemias.
&u( 3. Petechiae and ecchymosis are seen in throm/ocytopenia.
&c( C. Doss o position and "i/ratory sensation are common neurologic indings in pernicious anemia.
&u( 2. Cheilosis and !oilonychia are seen in iron deiciency anemia.
1,1. 2iagnosis/Hematology
A 6, year-old male presents with a normochromic) normocytic anemia and splenomegaly. His past history
re"eals se"eral episodes o /acterial pneumonia in the past year. The 03C count is '.),,, mm. with $%I
segmented neutrophils) .I /lasts) >,I mature lymphocytes) 1I /asophils) and 1I eosinophils. This most
li!ely represents
A. myelodysplastic syndrome.
3. acute lymphocytic leu!emia.
C. chronic lymphocytic leu!emia.
2. chronic myelogenous leu!emia.
&u( A. 6yelodysplastic syndrome may present with this clinical picture) /ut a C3C would indicate a normal or
reduced white cell count.
&u( 3. A hallmar! o acute leu!emia is pancytopenia with numerous circulating /lasts.
&c( C. Chronic lymphocytic leu!emia usually occurs ater the age o %, presenting with lymphocytosis O $,),,,
mm. and lymphocytes that appear mature.
&u( 2. Chronic myelogenous leu!emia presents with mar!edly ele"ated 03C count &mean 1%,),,,/mm.( with
let shited myeloid series.
1,$. 8cientiic Concepts/Pulmonology
0hich o the ollowing conditions will not produce a transudati"e pleural eusion1
A. Kaposi5s sarcoma
3. Pneumonia
C. Cirrhosis
2. 6esothelioma
&u( A. Kaposi5s sarcoma) pneumonia) or mesothelioma will produce a transudati"e pleural eusion.
&u( 3) 2. 8ee A or e7planation.
&c( C. Transudati"e pleural eusions result rom alteration in the ormation o pleural luid) the a/sorption o
pleural luid) or /oth) /y systemic actors. Docal actors aecting pleural luid a/sorption and/or ormation
produce e7udati"e pleural eusions. .
1,.. 8cientiic Concepts/<astrointestinal/+utritional
0hich o the ollowing pathophysiological processes is /elie"ed to initiate acute appendicitis1
A. ;/struction
3. Peroration
C. Hemorrhage
2. 4ascular compromise
&c( A. ;/struction o the appendiceal lumen /y lymphoid hyperplasia) a ecalith or oreign /ody initiates most
cases o appendicitis.
&u( 3) C) 2. 8ee A or e7planation.
1,'. 2iagnostic 8tudies/Cardiology
A $. year-old emale with history o palpitations presents or e"aluation. 8he admits to acute onset o rapid
heart /eating lasting seconds to minutes with associated shortness o /reath and chest pain. The patient states
she can relie"e her symptoms with "alsal"a. 0hich o the ollowing is the most appropriate diagnostic study to
esta/lish a deiniti"e diagnosis in this patient1
A. Cardiac catheteri9ation
3. Cardiac 6R*
C. Chest CT scan
2. Clectrophysiology study
&u( A. Cardiac catheteri9ation e"aluates coronary arteries /ut has no role in the diagnosis o supra"entricular
tachycardia.
&u( 3. Cardiac 6R* cannot diagnose and deine pathway o supra"entricular tachycardia.
&u( C. Chest CT scan will not esta/lish deiniti"e diagnosis o supra"entricular tachycardia.
&c( 2. Clectrophysiology study is useul in esta/lishing the diagnosis and pathway o comple7 arrhythmias such
as supra"entricular tachycardia.
1,%. 8cientiic Concepts/Cndocrinology
A !nown alcoholic presents to the emergency department with altered le"el o consciousness and a /lood
glucose le"el o .% mg/dD. 0hich o the ollowing /est e7plains this glucose result1
A. C7cess pancreatic insulin release
3. Rapid car/ohydrate discharge into the small /owel
C. Agonist insulin-receptor anti/ody ormation
2. Hepatic glycogen depletion and impaired gluconeogenesis
&u( A) 3) C. 8ee 2 or e7planation.
&c( 2. Alcohol-related hypoglycemia results rom hepatic glycogen depletion and impaired gluconeogenesis and
not due to anti/ody ormation) e7cessi"e insulin release rom the pancreas) or rapid release o car/ohydrate into
the small /owel.
1,6. Clinical Therapeutics/Cardiology
0hich o the ollowing is the chie ad"erse eect o thia9ide diuretics1
A. Hypo!alemia
3. Hypernatremia
C. Hypocalcemia
2. Hypermagnesemia
&c( A. Thia9ide diuretics can induce electrolyte changes. Principle among those is hypo!alemia.
&u( 3. Hyponatremia) not hypernatremia may /e a complication o thia9ide diuretics.
&u( C. Thia9ide diuretics cause the retention o calcium and would not cause hypocalcemia.
&u( 2. Thia9ide diuretics cause the retention o calcium and do not readily aect magnesium le"els.
1,>. History : Physical/*nectious 2iseases
The most distincti"e sign o pertussis is
A. stridor without cough.
3. producti"e cough with /asilar rales.
C. loose cough with coarse rhonchi.
2. paro7ysmal cough with crowing inspiration.
&u( A. 8tridor without cough suggests oreign /ody aspiration.
&u( 3. A producti"e cough and rales suggest pneumonia.
&u( C. Coarse rhonchi and a loose cough suggest /ronchitis.
&c( 2. A paro7ysmal cough with a loud inspiration &the whoop( is noted in pertussis.
1,-. History : Physical/*nectious 2iseases
The most distincti"e sign o pertussis is
A. stridor without cough.
3. producti"e cough with /asilar rales.
C. loose cough with coarse rhonchi.
2. paro7ysmal cough with crowing inspiration.

&u( A. 8tridor without cough suggests oreign /ody aspiration.
&u( 3. A producti"e cough and rales suggest pneumonia.
&u( C. Coarse rhonchi and a loose cough suggest /ronchitis.
&c( 2. A paro7ysmal cough with a loud inspiration &the whoop( is noted in pertussis.
1,B. History : Physical/Pulmonology
The inding o egophony is most consistent with
A. emphysema.
3. atelectasis.
C. pneumothora7.
2. lo/ar pneumonia.
&u( A. Cmphysema presents with diminished or a/sent /reath sounds and hyperresonance to percussion without
egophony.
&u( 3. Atelectasis most commonly has decreased /reath sounds and dullness to percussion without egophony.
&u( C. Pneumothora7 presents with a/sent /reath sounds) tactile remitus) and resonance to percussion without
egophony.
&c( 2. Cgophony occurs with consolidation caused /y lo/ar pneumonia.
11,. 2iagnosis/Arology/Renal
A patient presents with edema) which is most noticea/le in the hands and ace. Da/oratory indings include
proteinuria) hypoal/uminemia) and hyperlipidemia. The most li!ely diagnosis is
A. congesti"e heart ailure.
3. end-stage li"er disease.
C. nephrotic syndrome.
2. malnutrition.
&u( A. 2ependent edema is the most typical inding with CHE. Da/oratory indings do not generally include
proteinuria or hypoal/uminemia.
&u( 3. 8ymptoms o end-stage li"er disease usually include increased a/dominal girth indicating ascites.
Hypoal/uminemia can occur as a result o malnutrition or concurrently with nephrotic syndrome.
&c( C. Proteinuria) hyperlipidemia) and hypoal/uminemia are consistent with nephrotic syndrome.
&u( 2. 6alnutrition is mar!ed /y physical wasting) not edema. Hypoal/uminemia may /e seen) /ut
hyperlipidemia is not typical.
111. Clinical Therapeutics/C+T/;phthalmology
The /est course o action or a patient with a /othersome inlamed pingueculae &pingueculitis( is
A. anti/iotic drops.
3. e7cision.
C. 4isine drops.
2. no treatment.
&u( A. Anti/iotic drops ha"e no /eneit with pingueculitis.
&h( 3. C7cision is indicated or a pterygium that is threatening "ision.
&u( C. 4isine drops will not do anything) /ut artiicial tears may /e /eneicial.
&c( 2. 0ith pingueculitis) no treatment is necessaryF a short course o +8A*2 drops or steroids may help.
11$. Clinical Therapeutics/Pulmonology
An immunocompromised patient presents with signs and symptoms consistent with Degionella pneumophila
who has not responded to initial anti/iotic therapy with a macrolide. 0hich o the ollowing should /e added1
A. Clarithromycin &3ia7in(
3. Riampin &Riadin(
C. De"olo7acin &De"a@uin(
2. Amo7icillin-cla"ulanate &Augmentin(
&u( A. The macrolides &Clarithromycin( and luoro@uinolones &De"olo7acin( should /e used or initial
treatment) /ut not or adding to ailed treatments when a macrolide was already used.
&c( 3. Riampin should /e used as an ad=unct in patients with either a macrolide or @uinolone anti/iotic) who
ha"e ailed therapy) are immunocompromised or ha"e se"ere illness.
&u( C. 8ee A or e7planation.
&u( 2. Degionella pneumophila does not respond to 3eta-lactam anti/iotics.

11.. 2iagnosis/Psychiatry/3eha"ioral 6edicine
A mother /rings her 6 year-old /oy or e"aluation o school /eha"ior pro/lems. 8he says the teacher told her
that the /oy does not pay attention in class) that he gets up and runs around the room when the rest o the
children are listening to a story) and that he seems to /e easily distracted /y e"ents outside or in the hall. He
reuses to remain in his seat during class) and occasionally sits under his des! or crawls around under a ta/le.
The teacher told the mother this /eha"ior is interering with the child5s a/ility to unction in the classroom and
to learn. The mother states that she has noticed some o these /eha"iors at home) including his ina/ility to
watch his a"orite cartoon program all the way through. 0hich o the ollowing is the most li!ely diagnosis1
A. Antisocial disorder
3. 2ysthymic mood disorder
C. ;/sessi"e-compulsi"e disorder
2. Attention deicit hyperacti"ity disorder
&u( A. Antisocial /eha"ior disorder is characteri9ed /y disregard or rights o othersF a deect in the e7perience
o compunction or remorse or harming others.
&u( 3. 2ysthymic mood disorder is characteri9ed /y chronic) sad mood occurring or at least $ years in an adult
&one year in a child(. 3eha"ioral pro/lems are not part o this disorder.
&u( C. ;/sessi"e-compulsi"e disorder is characteri9ed /y recurrent o/sessions and compulsions that result in
an7iety and disrupti"e /eha"iors related to those compulsions.
&c( 2. Attention deicit hyperacti"ity disorder is characteri9ed /y inattention) including increased distracti/ility
and diiculty sustaining attentionF poor impulse control and decreased sel-inhi/itory capacityF and motor
o"eracti"ity and motor restlessness) which are per"asi"e and interere with the indi"idual5s a/ility to unction
under normal circumstances.
11'. Clinical *nter"ention/;rthopedics/Rheumatology
0hich o the ollowing is the treatment o choice or a torus &/uc!le( racture in"ol"ing the distal radius1
A. ;pen reduction and internal i7ation
3. Ace wrap or anterior splinting
C. Closed reduction and casting
2. Corticosteroid in=ection ollowed /y splinting
&u( A) C) 2. 8ee 3 or e7planation.
&c( 3. A torus or /uc!le racture occurs ater a minor all on the hand. These ractures are "ery sta/le and are
not as painul as unsta/le ractures. They heal une"entully in .-' wee!s.
11%. Clinical Therapeutics/Arology/Renal
0hich o the ollowing can /e used to treat chronic /acterial prostatitis1
A. Penicillin
3. Cephale7in &Kele7(
C. +itrourantoin &6acro/id(
2. De"olo7acin &De"a@uin(
C7planations
&u( A) 3) C. 8ee 2 or e7planation.
&c( 2. Chronic /acterial prostatitis &Type ** prostatitis( can /e diicult to treat and re@uires the use o
luoro@uinolones or trimethoprim-sulametho7a9ole) /oth o which penetrate the prostate.
116. Clinical *nter"ention/Cardiology
A $% year-old male with history o syncope presents or e"aluation. The patient admits to intermittent episodes
o rapid heart /eating that resol"e spontaneously. 1$ Dead CK< shows delta wa"es and a short PR inter"al.
0hich o the ollowing is the treatment o choice in this patient1
A. Radiore@uency catheter a/lation
3. 4erapamil &Calan(
C. Percutaneous coronary inter"ention
2. 2igo7in &Dano7in(
&c( A. Radiore@uency catheter a/lation is the treatment o choice on patients with accessory pathways) such as
0ol-Par!inson-0hite 8yndrome.
&h( 3. Calcium channel /loc!ers such as "erapamil decrease reractoriness o the accessory pathway or increase
that o the A4 node leading to aster "entricular rates) thereore calcium channel /loc!ers should /e a"oided in
patients with 0P0.
&u( C. Percutaneous coronary inter"ention is indicated in the treatment o coronary artery disease) not
pree7citation syndromes.
&h( 2. 2igo7in decreases reractoriness o the accessory pathway and increases that o the A4 node leading to
aster "entricular rates. *t should thereore /e a"oided in patients with 0P0.
11>. 8cientiic Concepts/Pulmonology
0hich o the ollowing pathophysiological processes is associated with chronic /ronchitis1
A. 2estruction o the lung parenchyma
3. 6ucous gland enlargement and go/let cell hyperplasia
C. 8mooth muscle hypertrophy in the large airways
2. *ncreased mucus adhesion secondary to reduction in the salt and water content o the mucus
&u( A. 2estruction o the gas-e7changing structures in the lung is characteristic o emphysema.
&c( 3. Chronic /ronchitis results rom the enlargement o mucous glands and go/let cell hypertrophy in the
large airways.
&u( C. There may /e smooth muscle hypertrophy in chronic /ronchitis /ut it is not to the e7tent as ound in
asthma and is not an underlying actor in the pathology o chronic /ronchitis.
&u( 2. A/normal a/sorption o sodium and a reduced rate o chloride secretion in cystic i/rosis leads to
thic!ening o the mucus and increase in adhesion o the mucus.
11-. Clinical Therapeutics/Psychiatry/3eha"ioral 6edicine
0hich o the ollowing dietary su/stances interact with monoamine o7idase-inhi/itor antidepressant drugs1
A. Dysine
3. <lycine
C. Tyramine
2. Phenylalanine
&u( A. Dysine) glycine) and phenylalanine are not !nown to interact with 6A; inhi/itors.
&u( 3) 2. 8ee A or e7planation.
&c( C. 6onoamine o7idase inhi/itors are associated with serious ood/drug and drug/drug interactions. Pt must
restrict inta!e o oods ha"ing a high tyramine content to a"oid serious reactions. Tyramine is a precursor to
norepinephrine.
11B. 8cientiic Concepts/<astrointestinal/+utritional
<allstones usually result in /iliary symptoms /y causing inlammation or o/struction ollowing migration into
the common /ile duct or
A. cystic duct.
3. pancreatic duct.
C. duodenal ampulla.
2. common hepatic duct.
&c( A. ;/struction o the cystic duct /y gallstones causes the typical symptom o /iliary colic. ;nce o/structed
the gall/ladder distends and /ecomes edematous and inlamed. <allstones can also migrate into the common
/ile duct through the cystic duct leading to a condition !nown as choledocholithiasis.
&u( 3. ;/struction o the pancreatic duct leads to de"elopment o acute pancreatitis.
&u( C. The duodenal ampulla is the area where the pancreatic duct and the common /ile duct empty into the
duodenum. <allstones do not cause o/struction at this distal site.
&u( 2. The common hepatic duct rom the li"er =oins the cystic duct rom the gall/ladder to orm the common
/ile duct. 8tone migration occurs along the pathway o the cystic duct to the common /ile duct) not along the
common hepatic duct.
1$,. Clinical *nter"ention/Pulmonology
An elderly patient with poorly-controlled Type $ dia/etes and renal disease de"elops a e"er o 1,$NE orally)
producti"e cough) and dyspnea. Physical e7amination demonstrates a respiratory rate o .$/min) la/ored
/reathing) and rales at the let /ase. Pulse o7imetry is B,I. 0hich o the ollowing is the ne7t appropriate step
in the management o this patient1
A. Administer ne/uli9ed corticosteroids
3. Admit to the hospital
C. ;ral antimicro/ial therapy
2. Cndotracheal intu/ation
&u( A. *nhaled corticosteroids are not utili9ed in the management o community-ac@uired pneumonia.
&c( 3. Community ac@uired pneumonia is the most deadly inectious disease in the A.8. *mportant ris! actors
or increased mor/idity and mortality include ad"anced age) alcoholism) comor/id medical conditions) altered
mental status) respiratory rate greater than ., /reaths/min) hypotension) and a 3A+ greater than .,.
&u( C. 2ue to the age o the patient) comor/id diseases) and current signs o respiratory distress) intra"enous not
oral antimicro/ial therapy is indicated.
&u( 2. Cndotracheal intu/ation is indicated or respiratory ailure unresponsi"e to conser"ati"e management.
1$1. 2iagnosis/Cndocrinology
A %. year-old emale who is well !nown to the practice presents to the oice complaining o increasing atigue)
constipation) and a weight gain o 1, l/ &'.% !g( o"er the past year. 8he also states others ha"e noticed a recent
hoarseness to her "oice) and she is /othered /y Gcharley horsesG in her legs that wa!e her up at night. Her past
medical history is unremar!a/le e7cept or a history o hyperthyroidism treated /y radioacti"e iodine % years
ago. 8he is currently ta!ing no medications and has no !nown drug allergies. 0hich o the ollowing is the most
li!ely cause o the patient5s symptoms1
A. Hypothyroidism
3. Hypoparathyroidism
C. 4ocal cord paralysis
2. Radiation thyroiditis
&c( A. The current symptoms) along with the past treatment o hyperthyroidism with radioacti"e iodine) would
indicate hypothyroidism.
&u( 3. This is a possi/le later complication o su/total thyroidectomy) not radioacti"e iodine therapy.
&u( C. This is an immediate complication o su/total thyroidectomy or in=ury and does not occur with
radioacti"e iodine therapy.
&u( 2. Radiation thyroiditis may occur ollowing radiation therapy /ut there is no history o the patient ha"ing
pre"ious e7ternal /eam radiation therapy.
1$$. History : Physical/Arology/Renal
0hich o the ollowing is most re@uently associated with /ladder cancer1
A. Hematuria
3. 2ysuria
C. Argency
2. Ere@uency
&c( A. 8igniicant persistent hematuria O. R3C/HPE on three urinalyses) a single urinalysis with O1,, R3C) or
gross hematuria) identiies signiicant renal or urologic lesions. 3ladder cancer usually presents with painless
hematuria.
&u( 3. 2ysuria) urgency) and re@uency are associated with irritati"e "oiding symptoms associated with cystitis.
&u( C) 2. 8ee 3 or e7planation.
1$.. History : Physical/<astrointestinal/+utritional
A . wee!-old male inant presents with recurrent regurgitation ater eeding that has progressed to pro=ectile
"omiting in the last ew days. The mother states that the child appears hungry all o the time. 8he denies any
diarrhea in the child. 0hich o the ollowing clinical indings is most li!ely1
A. 3ile-stained "omitus
3. Hemoccult positi"e stools
C. ;li"e-si9ed mass in the right upper a/domen
2. 8ausage-shaped mass in the upper-mid a/domen
&u( A. <astric o/struction) such as that seen with pyloric stenosis) causes "omiting that is not /ilious.
&u( 3. 3lood-strea!ed "omitus) /ut not hemoccult positi"e stools) may /e seen in pyloric stenosis.
&c( C. An oli"e-si9ed mass may /e palpated in the right upper a/domen in pyloric stenosis and i ound) is
pathognomonic or pyloric stenosis.
&u( 2. A sausage-shaped mass may /e noted in intussusception) not pyloric stenosis.
1$'. 2iagnosis/Psychiatry/3eha"ioral 6edicine
A mother /rings in her i"e year-old /oy or his school physical. 8he "oices some concerns a/out his readiness
or school) saying he seems to /e socially immature. 8he has noticed he does not interact with other children
well) and that when he plays with them) he has a tendency to Gplace themG and then run around them as i they
were statues. He rarely cries when he is hurt) and he shrugs o any attempt to hug him. He has good attention to
details) and will sit and draw the same geometric shapes o"er and o"er again) /ut does not seem interested in
learning the alpha/et. PHe a"oids eye contact with anyone. 0hich o the ollowing is the most li!ely diagnosis1
A. +ormal % year-old
3. 8ocial pho/ia
C. Autism
2. A"oidant personality
&u( A. This /eha"ior is not normal or a child this age.
&u( 3. 8ocial pho/ia is an e7cessi"e and persistent ear o social situations in which the person may /e
scrutini9ed /y others.
&c( C. Children with autism do not tend to ma!e eye contact and e"en a"oid it. They do not accept comort
when hurt and stien up when hugged. They do not tend to play with others and do not tend to imitate grown-
ups in play. They approach play in a more mechanical way using others as props rather than interacting w/
them.
&u( 2. A"oidant personality disorder is characteri9ed /y timidity) social aw!wardness) and a per"asi"e sense o
inade@uacy and ear o criticism.
1$%. Health 6aintenance/Pulmonology
A 6% year-old with C;P2 recei"ing their irst pneumococcal con=ugate "accination should /e re"accinated in
A. 1 year.
3. . years.
C. % years.
2. 1, years.
&u( A) 3) 2. 8ee C or e7planation.
&c( C. A single re"accination or a person o"er the age o 6% is recommended i it has /een more than % years
since they recei"ed their irst "accination.
1$6. Clinical Therapeutics/;rthopedics/Rheumatology
An -, year-old emale presents with pain in her "erte/ral column. Radiography re"eals compression racture o
T1$ that is consistent with osteoporotic compression racture. 0hich o the ollowing treatment modalities has
the potential to cause analgesia o the racture site with its use1
A. Calcitonin &6iacalcin( nasal spray
3. Alendronate &Eosama7(
C. Ralo7iene &C"ista(
2. Com/ined estrogen and progesterone &Prempro( therapy
&c( A. Calcitonin has the a/ility to cause analgesia when used or acute compression racture o the "erte/ral
/ody.
&u( 3. Alendronate is eecti"e in /uilding new /one or a patient with osteoporosis /ut has no associated
analgesic eect.
&u( C. Ralo7iene is a selecti"e estrogen receptor modulator and has positi"e eects on /one density when used
to treat osteoporosis. Ralo7iene) howe"er) has no analgesic properties.
&u( 2. Com/ined hormonal therapy may ha"e positi"e eects on /one density /ut it has no analgesic properties.
1$>. 8cientiic Concepts/2ermatology
0hich o the ollowing mediators is responsi/le or initiating the urticaric response1
A. Cyclic A6P
3. Prostaglandins
C. Prednisone
2. *gC
&u( A. *ncreased le"els o Cyclic A6P inhi/it the histamine response.
&u( 3. Prostaglandins inhi/it the release o histamine.
&u( C. Prednisone is used to treat urticaria.
&c( 2. *gC triggers the release o histamine rom mast cells that leads to urticaria.
1$-. Health 6aintenance/Hematology
0hich o the ollowing therapies is recommended or a 1. month-old child with sic!le cell disease1
A. Eolic acid and penicillin 4
3. Eerrous sulate and penicillin 4
C. Eolic acid and errous sulate
2. Eolic acid) errous sulate and penicillin 4
&c( A. Patients with sic!le cell disease should recei"e prophylactic penicillin 4 starting at $ months o age and
olic acid starting at 1 year o age. Eerrous sulate is not glo/ally recommended or patients with sic!le cell
disease.
&u( 3) C) 2. 8ee A or e7planation.
1$B. Clinical *nter"ention/Pulmonology
A patient with se"ere C;P2 presents to the C2 with a . day history o increasing shortness o /reath with
e7ertion and cough producti"e o purulent sputum. An arterial /lood gas re"eals a pH o >.$%) PaC;$ o >,
mmHg and Pa;$ o %, mmHg. He is started on al/uterol ne/uli9er) nasal o7ygen at $ liters per minute) and an
*4 is started. Ater one hour o treatment) his arterial /lood gas now re"eals a pH o >.1%) PaC;$ 1,, mmHg
and Pa;$ o >, mmHg. 0hich o the ollowing is the most appropriate ne7t step in his treatment1
A. 2ecrease the o7ygen low rate.
3. Administer oral corticosteroids.
C. *ntu/ate the patient.
2. Administer salmeterol &8ere"ent(
&h( A. 2ecreasing the ;$ low rate would /e harmul as it would decrease the amount o ;$ deli"ered to the pt.
&u( 3. Administration o steroids is an important treatment modality /ut this patient is in respiratory ailure and
needs more immediate therapy.
&c( C. This person has increasing respiratory ailure as indicated /y the raising PaC;$ le"els. *ntu/ation is
re@uired at this time.
&h( 2. Dong-acting /eta agonist therapy such as salmeterol is not utili9ed or rescue therapy.
1.,. 2iagnostic 8tudies/Pulmonology
A patient should /e tested or tu/erculosis prior to /eing treated with
A. etanercept &Cn/rel(.
3. cyclosporine &+eoral(.
C. methotre7ate &Rheumatre7(.
2. prednisone &2eltasone(.
&c( A. Ctanercept is an anti-cyto!ine agent used in the treatment o rheumatoid arthritis and has as a side eect
with potential or serious inections. ;ne o these side eects includes reacti"ation o dormant tu/erculosis.
&u( 3. Cyclosporine) methotre7ate) and prednisone do not ha"e the re@uirement to chec! or tu/erculosis prior
to initiating treatment.
&u( C) 2. 8ee 3 or e7planation.
1.1. Clinical Therapeutics/+eurology
0hich o the ollowing side eects is associated with long-term administration o phenytoin &2ilantin(1
A. Ata7ia
3. Hypotension
C. ;steomalacia
2. Cardiac dysrhythmia
&u( A. Ata7ia is associated most oten with acute oral o"erdosage o phenytoin.
&u( 3. Cardiac dysrhythmia) with or without hypotension) is an e7pected side eect o rapid *4 phenytoin
administration.
&c( C. ;steomalacia) or deminerali9ation o /one) is a side eect o phenytoin that may occur ater chronic
administration.
&u( 2. 8ee 3 or e7planation.
1.$. History : Physical/C+T/;phthalmology
Hairy leu!opla!ia has the greatest pre"alence o distri/ution on the
A. palate.
3. loor o the mouth.
C. lateral tongue.
2. gingi"a.
&u( A) 3) 2. 8ee C or e7planation.
&c( C. The lateral /order o the tongue is where hairy leu!opla!ia is commonly seen.
1... 2iagnosis/+eurology
A >% year-old male presents or a routine physical. 4itals are normal with no orthostatic changes. ;n physical
e7amination) a ine cortical mo"ement with repetiti"e ru//ing o the tip o the thum/ along the tips o the
ingers is noted at rest. 0hich o the ollowing is the most li!ely diagnosis1
A. 8ei9ure disorder
3. Peripheral neuropathy
C. 8hy-2rager syndrome
2. Par!inson5s disease
&u( A. 0hile a sei9ure may present with ine or gross uncontrolled motor mo"ements) the tremor descri/ed is
classic pill-rolling tremor noted in Par!inson5s disease.
&u( 3. Peripheral neuropathy presents with loss o sensation not tremor.
&u( C. 8hy-2rager syndrome is due to autonomic degeneration and typically presents with orthostatic
hypotension.
&c( 2. Par!inson5s disease presents with tremor at rest &pill-rolling() /rady!inesia) rigidity) and postural
1.'. Clinical Therapeutics/2ermatology
A '% year-old emale presents to the emergency department with generali9ed) hot) erythema o the s!in.
Physical e7am re"eals an oral temperature o 1,$ degrees Eahrenheit) purulent con=uncti"itis) and mucosal
erosions. Her s!in is painul and separates rom the dermis with touch. 0hich o the ollowing is the most li!ely
cause or this condition1
A. Ampicillin
3. Prednisolone
C. Aspirin
2. Hydrochlorothia9ide &HCTJ(
&c( A. 6edications are most re@uently implicated in to7ic epidermal necrolysis. These usually include)
analgesics &+8A*2s() anti/iotics &Ampicillin( and anticon"ulsants &Car/ama9epine(.
&u( 3. 8ystemic glucocorticoids may /e used early in the treatment o this condition and are not a cause.
&u( C. Aspirin is not lin!ed to to7ic epidermal necrolysis.
&u( 2. Hydrochlorothia9ide diuretics are not associated with the production o to7ic epidermal necrolysis.
1.%. Clinical *nter"ention/;/stetrics/<ynecology
To relie"e dependent edema in a pregnant patient) which o the ollowing should /e instituted as treatment1
A. Dimit luid inta!e.
3. Cle"ate the legs.
C. Prescri/e thia9ide diuretics.
2. 8trict a"oidance o sodium.
&u( A. Dimitation o luid is not indicated and may /e harmul.
&c( 3. 2ependent edema is a common and rarely serious complication o pregnancy due to impedance o "enous
return. Deg ele"ation impro"es circulation.
&u( C. Thia9ide diuretics are contraindicated and could /e harmul.
&u( 2. Cdema due to impedance o "enous return will not respond to sodium restriction.
1.6. History : Physical/<astrointestinal/+utritional
To urther assess ascites in a patient) the physician assistant instructs the patient to turn onto one side while
perorming percussion. 0hich o the ollowing is the reason or this maneu"er1
A. Testing or shiting o dullness on percussion
3. 8hiting o internal organs ma!ing percussion easier
C. Trying to elicit any pain while mo"ing
2. Trying to produce a caput medusa
&c( A. *n ascites) dullness shits to the more dependent side as the luid relocates into dependent space) while
tympany shits to the top as the gas-illed organs loat to the top o the ascitic luid.
&u( 3. 8ee A or e7planation.
&u( C. Pain with mo"ement is associated with peritonitis and not ascites.
&u( 2. Caput medusa is the dilation o the supericial a/dominal "eins due to increased intraa/dominal luid
accumulation. *t is "isi/le with the patient standing and does not need a special maneu"er or identiication.
1.>. 2iagnosis/Cndocrinology
A >-year-old child with a history o type 1 dia/etes mellitus or . years presents or routine ollow-up. The
mother states that the child has /een ha"ing nightmares and night sweats. Additionally) his a"erage morning
glucose readings ha"e risen rom an a"erage o 1,, mg/dD to 1'% mg/dD o"er the past wee!. This child is most
li!ely e7periencing
A. a growth spurt.
3. emotional pro/lems.
C. the 8omogyi eect.
2. the dawn phenomenon.
&u( A. +ightmares and night sweats are not associated with growth spurts.
&u( 3. 0ith this limited history) it is impossi/le to la/el the child as emotionally unsta/le.
&c( C. This reers to nocturnal hypoglycemia) which stimulates counter-regulatory hormone release resulting in
re/ound hyperglycemia.
&u( 2. This reers to an early morning rise in plasma glucose due to reduced tissue sensiti"ity to insulin /etween
% A6 and - A6. *t is not associated with nightmares and night sweats.
1.-. 8cientiic Concepts/+eurology
A patient complains o loss o sensation at the le"el o the um/ilicus. 0hich o the ollowing dermatomes is
aected1
A. T6
3. T-
C. T1,
2. T1$
&u( A)3) 2 8ee C or e7planation.
&c( C. The dermatome T1, is at the le"el o the um/ilicus.
1.B. Clinical Therapeutics/Pulmonology
0hich o the ollowing will result in decreased serum theophylline le"els in a patient with C;P21
A. Cimetidine
3. Congesti"e heart ailure
C. Cigarette smo!ing
2. Ciprolo7acin
&u( A) 3) 2. 8ee C or e7planation.
&c( C. Cigarette smo!ing will increase the hepatic clearance o theophylline) resulting in decreased le"els in the
system. The use o cimetidine or ciprolo7acin or the presence o congesti"e heart ailure will reduce hepatic
clearance and causing an increase in theophylline serum le"els.
1',. 2iagnosis/Psychiatry/3eha"ioral 6edicine
A $% year-old emale graduate student presents to the student health center or the eighth time in three wee!s to
/e sure she does not ha"e meningitis. 8he read that there was a student on campus who had meningitis last
month) and now she has headaches and is re@uesting to /e tested to ma!e sure she does not ha"e meningitis. 8he
has /een e"aluated at each "isit) and physical e7amination has /een completely normal each time. 0hich o the
ollowing is the most li!ely diagnosis1
A. Con"ersion disorder
3. Hypochondriasis
C. 6alingering
2. 8omati9ation disorder
C7planations
&u( A. Con"ersion disorder is characteri9ed /y onset o symptoms or deicits mimic!ing neurologic or medical
illness) /ut the etiology is psychological.
&c( 3. Hypochondriasis is the chronic preoccupation with the idea o ha"ing a serious disease) which is usually
not amena/le to reassurance
&u( C. 6alingering is the intentional production or eigning o physical or psychological signs and symptoms
or some gain.
&u( 2. 8omati9ation disorder is characteri9ed /y complaints o pain) oten related to gastrointestinal and se7ual
dysunction) and pseudoneurological symptoms.
1'1. 8cientiic Concepts/*nectious 2iseases
0hat is the mechanism or the relapsing e"ers associated with malaria1
A. Release o malarial mero9oites into the /loodstream
3. Release o to7ins rom Plasmodium species
C. Attachment o Plasmodium species to receptor sites on the red /lood cells
2. *n"asion o hepatocytes /y Plasmodium species
&c( A. Ee"er) chills) and sweats coincide with the release o mero9oites rom red /lood cells that ha"e /een
inected with Plasmodium species. Cach mero9oite may inect a new red /lood cell) leading to a cycle o
in"asion growth) and release.
&u( 3) C) 2. 8ee A or e7planation.
1'$. Clinical *nter"ention/Hematology
A $B year-old patient with idiopathic throm/ocytopenia purpura &*TP( is treated with prednisone therapy.
2espite therapy) platelet counts remain consistently /elow $,),,,/microliter o"er the course o 6 wee!s. 0hich
o the ollowing is the most appropriate inter"ention or this patient1
A. Aspirin
3. *ntra"enous immunoglo/ulin
C. 2ana9ol &2anocrine(
2. 8plenectomy
&h( A. Aspirin inhi/its platelet unction and could lead to signiicant /leeding and death o this patient.
&u( 3. *ntra"enous immunoglo/ulin can /e utili9ed or short-term treatment) /ut the platelet count is li!ely to
return to /aseline within a month.
&u( C. 2ana9ol is typically reser"ed or *TP that ails to respond to splenectomy.
&c( 2. Persistently low platelet counts &M $,),,,( re@uire eecti"e long-term treatment) and splenectomy is the
treatment o choice.
1'.. 2iagnostic 8tudies/+eurology
A $> year-old emale presents to your oice or e"aluation o wea!ness) "isual loss) and sensory loss o"er the
right great toe. These symptoms ha"e occurred during three episodes appro7imately three months apart with
each episode lasting a/out three days. 0hich o the ollowing tests would /e most useul in urther e"aluating
this patient1
A. 6R* o the /rain
3. Clectromyograph
C. <lucose tolerance test
2. Clectroencephalograph
&c( A. 6ultiple sclerosis typically presents with relapsing wea!ness o the lim/s) sensory loss) paresthesias) and
"isual changes. 2iagnosis is /ased on history and either a/normal /rain or spinal cord 6R*) or "isual)
auditory) or somatosensory e"o!ed electrical response.
&u( 3. 8ee A or e7planation.
&u( C. <lucose tolerance test is used in the e"aluation o dia/etes mellitus.
&u( 2. Clectroencephalograph is used to e"aluate patients or possi/le sei9ure disorder.
1''. Clinical Therapeutics/Pulmonology
A patient ta!ing /leomycin &3leno7ane( should /e monitored or which o the ollowing side eects1
A. ;ptic neuritis
3. Hyperuricemia
C. Cncephalopathy
2. Pulmonary i/rosis
&u( A. ;ptic neuritis is a potential side eect o etham/utol) used in the treatment o tu/erculosis.
&u( 3. Hyperuricemia or encephalopathy are not !nown side eects o /leomycin.
&u( C. 8ee 3 or e7planation.
&c( 2. Pulmonary i/rosis and pulmonary iniltrates are !nown side eects o /leomycin.
1'%. History : Physical/Cardiology
A patient presents or a ollow-up "isit or chronic hypertension. 0hich o the ollowing indings may /e noted
on the undoscopic e7amination o this patient1
A. cherry-red o"ea
3. /o7car segmentation o retinal "eins
C. papilledema
2. arterio"enous nic!ing
&u( A. Cherry-red o"ea and /o7car segmentation o the retinal "eins are indings seen in central retinal artery
occlusion.
&u( 3. 8ee letter A or e7planation.
&u( C. Papilledema is noted in conditions causing increased intracranial pressure.
&c( 2. Arterio"enous nic!ing is common in chronic hypertension.
1'6. History : Physical/<astrointestinal/+utritional
0hich o the ollowing clinical indings would /e seen in a patient with ood poisoning caused /y
8taphylococcus aureus1
A. *ngestion o mayonnaise-/ased salads '- hours earlier
3. 3loody diarrhea with mucus or one wee!
C. A/dominal cramps and "omiting or '- hours
2. High e"er or 1 wee!
&u( A. A preormed to7in causes staphylococcal ood poisoningF it has a short incu/ation period o 1-- hours.
&u( 3. 3ecause 8taphylococcus aureus does not in"ade the mucus) /lood and mucus are not seen with this
noninlammatory cause o ood poisoning.
&c( C. A/dominal cramps) nausea) "omiting) and watery diarrhea typically last 1-$ days with staphylococcal
ood poisoning.
&u( 2. 8taphylococcal ood poisoning may /e associated with low-grade e"er or su/normal temperature.
1'>. 2iagnostic 8tudies/Cardiology
0hich o the ollowing diagnostic tests should /e ordered initially to e"aluate or suspected deep "enous
throm/osis o the leg1
A. 4enogram
3. Arteriogram
C. 2uple7 ultrasound
2. *mpedance plethysmography
&u( A. 4enogram has /een replaced /y nonin"asi"e tests due to discomort) cost) technical diiculties) and
complications) such as phle/itis.
&h( 3. Throm/ophle/itis is a "enous pro/lem) not an arterial one. Any unnecessary in"asi"e procedure is
potentially harmul.
&c( C. Altrasound is the techni@ue o choice to detect deep "enous throm/osis in the leg.
&a( 2. *mpedance plethysmography is e@ui"alent to ultrasound in detecting throm/i o the emoral and popliteal
"eins) /ut it may miss early) nonocclusi"e throm/i.
1'-. Clinical *nter"ention/Cndocrinology
A patient presents with signs and symptoms o Cushing5s syndrome. C7tensi"e diagnostic e"aluation re"eals an
ACTH-secreting pituitary adenoma. Eirst-line therapy should consist o
A. pituitary radiation.
3. medical adrenalectomy.
C. transsphenoidal resection o the tumor.
2. amiloride &6idamor(.
&u( A) 3) 2. 8ee C or e7planation.
&c( C. Transsphenoidal resection o the tumor cures a/out -,I o patients. The remainder can /e gi"en a
com/ination o pituitary radiation and medical adrenalectomy with one or more drugs. * these procedures ail)
the last option is /ilateral adrenalectomy.
1'B. Clinical Therapeutics/Pulmonology
A $6 year-old man is stung /y a /ee) and shortly thereater) a wheal de"elops at the site o the sting. He soon
eels lushed and de"elops hi"es) rhinorrhea) and tightness in the chest. He is seen in the urgent care center.
*mmediate therapy should /e to
A. transer him to a local hospital emergency department.
3. apply a cold compress to site o the sting.
C. administer su/cutaneous epinephrine.
2. administer oral al/uterol.
&u( A. 8ystemic &anaphylactic( reactions can rapidly /ecome lie-threatening. 2elay in t7 may cause death.
&u( 3. This is only supporti"e local therapy and does not address the need to treat the systemic reaction present.
&c( C. Cpinephrine hydrochloride 1#1,,,) ,.$ to ,.% mD su/cutaneously is indicated or the initial treatment o
this systemic reaction. Additional in=ections may /e gi"en e"ery $, to ., minutes i needed.
&u( 2. Al/uterol is indicated in the presence o /ronchospasm &suggested /y the presence o chest tightness() /ut
would /e deli"ered /y an aerosol) not an oral) route.
1%,. 2iagnosis/C+T/;phthalmology
An 1- year-old se7ually acti"e emale was seen in the student health clinic 1 wee! ago or a sore throat. A
streptococcal antigen test was positi"e) and she was gi"en a prescription or oral penicillin. Ater . days) she
stopped her medication /ecause she elt /etter. 8he now presents with a se"ere sore throat. ;n physical
e7amination) she has a temperature o 1,$.6N E &.B.$N C() mar!ed pharyngeal erythema) medial de"iation o the
sot palate on the let) Ptender let anterior cer"ical adenopathy) and a Ghot potatoG "oice. The rest o her history
and physical e7amination are unremar!a/le. 0hich o the ollowing is the most li!ely diagnosis1
A. Recurrent streptococcal pharyngitis
3. *nectious mononucleosis
C. <onococcal pharyngitis
2. Peritonsillar a/scess
&u( A. This presentation suggests a complication o an incompletely treated streptococcal pharyngitis rather
than recurrent disease.
&u( 3. *nectious mononucleosis may present with se"ere sore throat) e"er) and cer"ical adenopathy in this age
group) /ut would not cause de"iation o the sot palate or the muled "oice.
&u( C. <onococcal pharyngitis usually ollows a more indolent course than this patient5s presentation.
&c( 2. The sot palate de"iation and a muled "oice are classic signs o peritonsillar a/scess.
1%1. 0hich o the ollowing would pro"ide the most speciic inormation regarding the unctional cardiac status
in a patient with chronic heart ailure1
A. Clectrocardiogram
3. Chest 7-ray
C. 8erum electrolytes
2. Cchocardiogram
&u( A. Clectrocardiogram oers no speciic ino o unctional status) /ut may pro"ide clues a/out the cause.
&u( 3. A chest 7-ray may show indings o chronic heart ailure) such as cardiomegaly or pulmonary congestion)
/ut does not relect cardiac unctional status.
&u( C. 8erum electrolytes may /e a/normal) either as a result o heart ailure) or as a contri/uting actor) /ut
they do not indicate unctional status.
&c( 2. Cchocardiogram will estimate e=ection raction) which is an indicator o let "entricular unction.
1%$. 2iagnostic 8tudies/;/stetrics/<ynecology
A .. year-old emale presents or ollow-up o her Pap smear that showed cer"ical dysplasia. 0hich o the
ollowing is the most appropriate diagnostic procedure1
A. Cone /iopsy
3. Aspiration needle /iopsy
C. 2ilation and curettage
2. Colposcopy-directed /iopsy
C7planations
&u( A. A cer"ical cone /iopsy may /e indicated in urther e"aluation o this patient) /ut it is dependent on the
results o the colposcopy.
&u( 3. An aspiration needle /iopsy has no role in the e"aluation o cer"ical dysplasia.
&u( C. 2ilatation and curettage has no role in either the diagnosis or treatment o isolated cer"ical dysplasia.
&c( 2. A colposcopy-directed /iopsy is the irst diagnostic e"aluation indicated or cer"ical dysplasia.
1%.. 2iagnostic 8tudies/;/stetrics/<ynecology
A decrease in the etal heart rate &EHR( occurring late during contractions is noted. The EHR returns to the
/aseline slowly ater the uterine contraction. The physician assistant should /e alerted to the possi/ility o
A. pel"ic dystocia.
3. precipitous la/or.
C. etal head compression.
2. placental insuiciency.
&u( A. Pel"ic dystocia) particularly that due to small /ony architecture) is the most common cause o passage
a/normalities and is not directly associated with EHR decelerations.
&u( 3. This reers to the length o la/or) not decelerations in EHR.
&u( C. The drop in EHR is caused /y an intererence with uterine /lood low to the inter"illous space causing an
early) not late) deceleration.
&c( 2. Placental insuiciency is the pro/a/le cause o etal distress resulting in late decelerations.
1%'. 2iagnosis/Cardiology
A .6 year-old patient with cardiomyopathy secondary to "iral myocarditis de"elops atigue) increasing dyspnea)
and lower e7tremity edema o"er the past . days. He denies e"er. A chest 7-ray shows no signiicant increase in
heart si9e) /ut re"eals prominence o the superior pulmonary "essels. 3ased on these clinical indings) which o
the ollowing is the most li!ely diagnosis1
A. Heart ailure
3. 8u/acute /acterial endocarditis
C. Pulmonary em/olus
2. Pneumonia
&c( A. <i"en the presence o cardiomyopathy) the patient5s heart has decreased unctional reser"e. The
symptoms and chest 7-ray indings are typical o congesti"e heart ailure.
&u( 3. Cndocarditis occurs as a result o inection that primarily occurs in the /lood stream. Cndocarditis would
present with signs o inection or seeding rather than signs o heart ailure.
&u( C. Pulmonary em/olus usually presents with an acute onset o chest pain) se"ere dyspnea) and an7iety.
&u( 2. Pneumonia is less li!ely since there is no e"er and edema is not usually associated with pneumonia.
1%%. Clinical *nter"ention/Cardiology
0hich o the ollowing is irst-line t7 or symptomatic /radyarrhythmias due to sic! sinus syndrome &888(1
A. Permanent pacema!er
3. Radiore@uency a/lation
C. Antiarrhythmics
2. Anticoagulation therapy
&c( A. Permanent pacema!ers are the therapy o choice in patients with symptomatic /radyarrhythmias in sic!
sinus syndrome.
&u( 3. Radiore@uency a/lation is used or the treatment o accessory pathways in the heart.
&u( C) 2. 8ee A or e7planation.
1%6. 2iagnosis/<astrointestinal/+utritional
A '$ year-old male with a history o constipation presents with complaints o se"ere pain with deecation
descri/ed as eeling li!e he is Gtearing apart.G He has also noted occasional small amounts o /lood on toilet
paper. C7ternal e7amination o the rectum is unremar!a/le and an internal rectal e7am cannot /e perormed due
to se"ere pain when attempted. 0hich o the ollowing is the most li!ely diagnosis1
A. Proctitis
3. Anal issure
C. Rectal prolapse
2. *nternal hemorrhoids
C7planations
&u( A. Proctitis is usually caused /y anorectal inections that produce symptoms o anorectal discomort)
tenesmus) constipation) and discharge rom the rectum.
&c( 3. Anal issures are easily diagnosed rom history alone with the classic inding o se"ere pain upon
deecation. Constipation is also a common cause o the trauma that leads to de"elopment o a issure.
&u( C. Rectal prolapse is commonly seen in elderly emales with complaints that include an anal mass) rectal
/leeding and a change in /owel ha/its.
&u( 2. 0hile internal hemorrhoids may cause rectal /leeding) tearing pain is an uncommon complaint unless
there is e"idence o throm/osis o irreduci/le tissue.
1%>. Clinical Therapeutics/+eurology
0hich o the ollowing drugs is the irst choice or insomnia in an elderly patient1
A. 3ar/iturates
3. Cholinesterase inhi/itors
C. 3en9odia9epines
2. 3eta-/loc!ers
&u( A. The use o /ar/iturates is o/solete and these agents may actually disrupt the sleep cycle.
&u( 3. Cholinesterase inhi/itors are used in the treatment o Al9heimer5s) /ut ha"e no eect on insomnia.
&c( C. 3en9odia9epines are the drugs o choice or insomnia in the elderly population.
&u( 2. 3eta-/loc!ers ha"e no use in the treatment o insomnia.
1%-. History : Physical/Cardiology
0hat type o chest pain is most commonly associated with a dissecting aortic aneurysm1
A. 8@uee9ing
3. 2ull) aching
C. Ripping) tearing
2. 3urning
&u( A. 8@uee9ing pain is more characteristic o angina or esophageal pain.
&u( 3. 2ull) aching pain is more characteristic o chest wall pain) possi/ly angina) or an7iety.
&c( C. A dissecting aortic aneurysm oten presents with a "ery se"ere ripping) tearing-li!e pain.
&u( 2. 3urning pain is more characteristic o esophageal relu7) esophagitis) or tracheo/ronchitis.
1%B. 2iagnostic 8tudies/;rthopedics/Rheumatology
0hich o the ollowing "iews on plain ilms is preerred to identiy spondylolysis1
A. Anterior
3. Posterior
C. ;/li@ue
2. Dateral
&u( A) 3) 2. 8ee C or e7planation.
&c( C. The deect in the pars articularis &usually /ilateral( is /est "isuali9ed on the o/li@ue pro=ections on plain
ilms.
16,. Clinical *nter"ention/;rthopedics/Rheumatology
A 1% year-old sot/all player presents ater =amming the distal tip o her inger into se"ere le7ion. 8he is una/le
to e7tend the distal phalan7 and she has pain on palpation o the distal interphalangeal =oint. L-ray o the hand
ails to re"eal any associated a"ulsion racture. 0hich o the ollowing is the treatment o choice1
A. ;pen reduction and internal i7ation
3. Continuous e7tension o the 2*P with splinting
C. Continuous le7ion o the P*P with splinting
2. Application o short arm cast
&u( A) C. 8ee 3 or e7planation.
&c( 3. The treatment o choice or a tear in the e7tensor tendon o the inger is continuous e7tension o the 2*P
"ia splinting or 6 to - wee!s.
&u( 2. 8hort arm casting is indicated in wrist and metacarpal in=uries /ut not in 2*P e7tensor in=uries.
161. 2iagnosis/2ermatology
A pt presents with loss o pigmentation on the /ac! o hands) ace) and /ody olds due to the a/sence o
epidermal melanocytes. There has /een impro"ement with PA4A t7. 0hich o the ollowing is the most li!ely
d71
A. Pityriasis al/a
3. Tinea "ersicolor
C. 4itiligo
2. 6elasma
&u( A. Pityriasis al/a is caused /y dermal inlammation that /ecomes scaly and hypopigmented.
&u( 3. Tinea "ersicolor is a ungal inection o the s!in presenting as o"al or circular lesions
&c( C. 4itiligo is the ac@uired loss o pigmentation due to the a/sence o epidermal melanocytes presenting on
the /ac! o hands) ace) or /ody olds.
&u( 2. 6elasma is an ac@uired /rown hyperpigmentation in"ol"ing the ace and nec! in women during their
second or third trimester o pregnancy.
16$. 8cientiic Concepts/Psychiatry/3eha"ioral 6edicine
0hich o the ollowing medications used in the management o an7iety has a delayed onset o action1
A. /uspirone &3u8par(
3. diphenhydramine &3enadryl(
C. lora9epam &Ati"an(
2. /utal/ital &Eiorinal(
C7planations
&c( A. 3uspirone ta!es se"eral days to wee!s or it to ha"e clinical acti"ity.
&u( 3. 2iphenhydramine wor!s as a histamine /loc!er and will cause sedation immediately /ecause o its
anticholinergic eects.
&u( C. Dora9epam is an an7iolytic medication that has an immediate onset o acti"ity.
&u( 2. 3utal/ital is a short to intermediate-acting /ar/iturate that has immediate acti"ity.
16.. Clinical *nter"ention/C+T/;phthalmology
A %> year-old male was wor!ing on his arm) when some manure was slung hitting his let eye. He presents
se"eral days ater with a red) tearing) painul eye. Eluorescein stain re"eals upta!e o"er the cornea loo!ing li!e a
shallow crater. 0hich o the ollowing inter"entions would /e harmul1
A. ;phthalmic anti/iotics
3. Pressure patch
C. C7amination or "isual acuity
2. Copious irrigation
&u( A. ;phthalmic anti/iotics and copious irrigation are indicated when treating a patient with a suspected
corneal ulcer due to an inectious cause.
&c( 3. Patching o the eye ater a/rasion associated with organic material contamination is contraindicated due
to increased ris! o ungal inection.
&u( C. C7amination or assessment o "isual acuity should /e perormed.
&u( 2. 8ee A or e7planation.
16'. History : Physical/;rthopedics/Rheumatology
A >% yo emale alls on her outstretched arm. 8he sustains a humeral mid-shat racture. +er"e impingement
occurs due to the racture. 0hat is the most li!ely physical e7amination a/normality that will /e encountered1
A. *na/ility to e7tend the wrist against resistance
3. +um/ness o"er the deltoid muscle in the shoulder
C. 0inging o the scapula
2. 0ea!ness o the rotator cu
&c( A. The radial ner"e is most li!ely entrapped /y this racture. Radial ner"e damage will cause an ina/ility to
e7tend the wrist against resistance.
&u( 3. A7illary ner"e in=ury results in num/ness o"er the deltoid muscleF this ner"e is more commonly in=ured
in pro7imal humeral ractures and anterior shoulder dislocations.
&u( C. *n=ury to the long thoracic ner"e causes winging o the scapula due to its inner"ation o the serratus
anterior muscle.
&u( 2. *n=ury to the su/scapular ner"e results in wea!ness and pain o the inraspinatus muscleF this in=ury is
commonly seen in "olley/all players rom repetiti"e stress.
16%. Clinical *nter"ention/C+T/;phthalmology
A 16 year-old male in"ol"ed in a ight sustained a laceration to his right upper eyelid. He is una/le to open his
eye) and a possi/le laceration o the glo/e is suspected. 0hich o the ollowing is the ne7t step1
A. Ase a slit lamp to determine the e7tent o the in=ury.
3. Ase luorescein strips to determine the e7tent o in=ury.
C. Apply a metal eye shield and reer to an ophthalmologist.
2. Apply anti/iotic ointment to the lid and rechec! in $' hours.
&h( A) 3) 2. 8ee C or e7planation.
&c( C. Protect the eye rom any pressure with a rigid metal eye shield and reer or immediate ophthalmologic
consultation. A"oid unnecessary actions that would delay treatment or cause urther in=ury.
166. 2iagnostic 8tudies/<astrointestinal/+utritional
0hich o the ollowing typical indings would /e re"ealed during a sigmoidoscopy on a patient with Crohn5s
disease o the intestine1
A. Rectal pseudopolyps
3. 2iuse ulceration and /leeding
C. 8heets o 03Cs with inlamed mucosa
2. *ntermittent longitudinal mucosal ulcers and issures
&u( A. Rectal pseudopolyps are associated with ulcerati"e colitis rather than Crohn5s.
&u( 3. 2iuse ulcerations and /leeding are more characteristic o ulcerati"e colitis than Crohn5s disease.
&u( C. 8heets o 03Cs or Gpseudomem/ranesG can /e detected in patients with pseudomem/ranous colitis.
&c( 2. Alcerations tend to /e linear with trans"erse issures in Crohn5s disease. These s!ip lesions are common
with Crohn5s disease.
16>. 2iagnosis/+eurology
A >$ year-old patient with a history o hypertension and atrial i/rillation presents with episodes o wea!ness)
num/ness) and paresthesias in the right arm. At the same time) she notes speech diiculty and loss o "ision in
her let eye. These symptoms come on a/ruptly and clear within minutes. Physical e7amination is normal
e7cept or the pre"iously !nown arrhythmia. 0hich o the ollowing is the most li!ely diagnosis1
A. Eocal sei9ure
3. 6igraine headache
C. Hypoglycemic episodes
2. Transient ischemic attac!
&u( A. Eocal sei9ures usually cause a/normal motor mo"ement rather than wea!ness or loss o eeling.
&u( 3. Patients with migraines commonly ha"e a history o episodes since adolescence.
&u( C. Hypoglycemic episodes do not present with ocal neurological indings.
&c( 2. This patient5s symptoms are consistent with transient ischemia in the carotid territory. Atrial i/rillation is
a ris! actor or cere/ral em/oli.
16-. 8cientiic Concepts/Hematology
A deect in which o the ollowing physiologic processes can lead to the de"elopment o a microcytic)
hypochromic anemia1
A. Hemoglo/in production
3. *g< anti/ody ormation
C. Red /lood cell nuclear maturation
2. Red /lood cell mem/rane deects
&c( A. A deiciency in su/stances re@uired or hemoglo/in synthesis result in R3Cs with a deicient mean
corpuscular hemoglo/in concentration and usually microcytosis.
&u( 3. *g< anti/ody ormation would led to the de"elopment o a hemolytic anemia.
&u( C. Alterations in red /lood cell nuclear maturation led to the de"elopment o macrocytic red /lood cells.
&u( 2. Red /lood cell mem/rane deects led to the de"elopment o hemolytic anemia.
16B. Health 6aintenance/Cardiology
A %$ year-old o/ese emale with a history o hypertension) to/acco a/use) and hyperlipidemia presents or
routine ollow-up. 0hich o her ris! actors or coronary atherosclerosis is not modiia/le1 P
A. Age
3. High D2D
C. Hypertension
2. ;/esity
&c( A. Age is a non modiia/le ris! actor) as is amily history o premature coronary heart disease
&u( 3. High D2D is a modiia/le ris! actor) as is HT+) low H2D) o/esity) to/acco a/use) physical inacti"ity
&u( C) 2. 8ee 3 or e7planation.
1>,. 2iagnosis/Pulmonology
0hich o the ollowing physical e7amination indings would /e consistent with a pleural eusion1
A. Hyperresonance to percussion
3. *ncreased tactile remitus
C. Anilateral lag on chest e7pansion
2. Cgophony
&u( A. Hyperresonance to percussion would /e suggesti"e o emphysema or pneumothora7.
&u( 3. *ncreased tactile remitus would /e consistent with a consolidation.
&c( C. A lag on chest e7pansion may /e seen in the presence o a pleural eusion.
&u( 2. The presence o egophony would /e consistent with a consolidation.
1>1. Clinical Therapeutics/;rthopedics/Rheumatology
A %% year-old emale presents with complaints o stiness) aching) and pain in the muscles o her nec!)
shoulders) lower /ac!) hips) and thighs. There is no associated wea!ness associated with the stiness and
achiness. Da/oratory e"aluation shows an ele"ated C reacti"e protein and erythrocyte sedimentation rate. 0hich
o the ollowing medications is used to treat this condition immediately and will also ser"e to pre"ent a !nown
complication rom this disorder1
A. <lucocorticoids
3. Cyclophosphamide &Cyto7an(
C. 6ethotre7ate &Rheumatre7(
2. A9athioprine &*muran(

&c( A. This patient has polymyalgia rheumatica and treatment with glucocorticoids can relie"e discomort and
pre"ent the associated ischemic temporal arteritis) which threatens "ision.
&u( 3. Cyclophosphamide is an immunosuppressant used in the treatment o acute leu!emia.
&u( C. 6ethotre7ate is a olate inhi/itor used to treat rheumatoid arthritis) not polymyalgia rheumatica.
&u( 2. A9athioprine is an immunosuppressant that is used to treat rheumtatic disease and inlammatory /owel
disease) not polymyalgia rheumatica.
1>$. Clinical Therapeutics/Arology/Renal
A .% year-old pregnant patient presents with e"er) chills) and let-sided lan! pain. ;n physical e7amination
let-sided C4A tenderness is noted. Arinalysis re"eals numerous white /lood cells and white /lood cell casts.
0hich o the ollowing is the most appropriate treatment1
A. ;ral ciprolo7acin &Cipro(
3. ;ral trimethoprim-sulametho7a9ole &3actrim(
C. *4 gentamicin &<aramycin(
2. *4 cetria7one &Rocephin(
&h( A. 8ee 3 or e7planation.
&h( 3. The luoro@uinolones and trimethoprim-sulametho7a9ole are contraindicated in pregnancy.
&u( C. <entamicin is not indicated as irst line therapy in the treatment o pyelonephritis in a pregnant patient.
&c( 2. *4 cephalosporins are irst line treatment o pyelonephritis in a pregnant patient) ollowed /y oral step-
down therapy.
1>.. Clinical *nter"ention/Psychiatry/3eha"ioral 6edicine
A $6 year-old emale arri"es in the emergency department with riends who say she was standing in ront o her
church) dressed in a white /athro/e) claiming to /e the 4irgin 6ary and handing out Q1,, /ills to all passers-
/y. Her riends noted that she had /een depressed lately) /ut now seems completely euphoric. 8he had a similar
episode two years ago. 0hich o the ollowing is the most appropriate treatment1
A. *npatient olan9apine &Jypre7a( therapy
3. *npatient electrocon"ulsi"e therapy
C. ;utpatient paro7etine &Pa7il( therapy
2. ;utpatient psychotherapy
&c( A. T7 o the manic phase is usually done in the hospital to protect patients rom /eha"iors associated with
grandiosity &spending inordinate amounts o money) ma!ing em/arrassing speeches) etc.(. Dithium) "alproate)
and olan9apine are considered eecti"e in the manic stageF the depressi"e stage is treated with antidepressants.
&u( 3) C) 2. 8ee A or e7planation.
1>'. 2iagnostic 8tudies/Arology/Renal
A 1B year-old patient was in"ol"ed in a motor "ehicle crash and /rought to the emergency department ully
immo/ili9ed. The patient sustained multiple /lunt in=uries to the chest and a/domen. 2uring the trauma
assessment) there was no /lood at the urethral meatus and a Eoley catheter was placed. The urine was positi"e
or /lood on the dipstic!. 0hich o the ollowing is the most appropriate diagnostic test1
A. Retrograde urethrography
3. CT scan o a/domen and pel"is
C. 8erum haptoglo/in
2. Arine myoglo/in
&u( A. A retrograde urethrogram should /e perormed when /lood is ound at the e7ternal urinary meatus prior
to insertion o a catheter.
&c( 3. CT scan o the a/domen and pel"is is indicated in /lunt trauma including those resulting in hematuria or
when renal in=ury is suspected.
&u( C. A decreased serum haptoglo/in is seen in hemolysis and does not pro"ide inormation on renal status.
&u( 2. A positi"e test or /lood in the a/sence o red /lood cells on urine e7amination suggests myoglo/inuria)
and should /e conirmed /y electrophoresis.
1>%. Clinical Therapeutics/<astrointestinal/+utritional
A '- year-old male presents with complaints o heart/urn that occurs appro7imately '% minutes ater eating
a/out three times a wee! that is relie"ed /y antacids. He claims to ha"e ollowed ad"ice a/out ele"ating the
head o the /ed) a"oiding spicy oods) and losing weight) /ut continues to ha"e heart/urn. 0hich o the
ollowing is the most appropriate ne7t step1
A. Ranitidine &Jantac(
3. 8ucralate &Caraate(
C. 6etoclopramide &Reglan(
2. 6isoprostol &Cytotec(
&c( A. Ranitidine) an H$ receptor /loc!er) is indicated or the treatment o mild) intermittent symptoms o
gastroesophageal relu7 disease.
&u( 3. 8ucralate is used in the treatment o duodenal ulcers.
&u( C. 6etoclopramide is indicated or the treatment o gastroparesis as a irst-line agent and as a second-line
agent in the treatment o reractory gastroesophageal relu7.
&u( 2. 6isoprostol is indicated or the pre"ention o +8A*2-induced gastritis.
1>6. Clinical *nter"ention/Cndocrinology
Radioacti"e iodine &*1.1( is most successul in treating hyperthyroidism that results rom
A. <ra"e5s disease.
3. su/acute thyroiditis.
C. Hashimoto5s thyroiditis.
2. papillary thyroid carcinoma.
&c( A. Radioacti"e iodine &*1.1( is an e7cellent method to destroy o"eracti"e thyroid tissue o <ra"e5s disease.
&u( 3. Radioacti"e iodine is ineecti"e in su/acute thyroiditis due to the thyroid5s low upta!e o iodine.
&u( C. Radioiodine upta!e is low in Hashimoto5s thyroiditis ma!ing radioacti"e therapy ineecti"e.
&u( 2. Papillary thyroid carcinoma is a common thyroid malignancy and should /e treated /y a thyroidectomy.
1>>. 2iagnosis/Cardiology
An - year-old /oy is /rought to a health care pro"ider complaining o dyspnea and atigue. ;n physical
e7amination) a continuous machinery murmur is heard /est in the second let intercostal space and is widely
transmitted o"er the precordium. The most li!ely diagnosis is
A. "entricular septal deect.
3. atrial septal deect.
C. congenital aortic stenosis.
2. patent ductus arteriosus.
&u( A. 4entricular septal deect causes a holosystolic murmur rather than a continuous machinery-li!e murmur.
&u( 3. Atrial septal deect causes a i7ed split 8$ rather than a continuous systolic heart murmur.
&u( C. Congenital aortic stenosis causes a crescendo-decrescendo systolic murmur heard /est in the second
intercostal space.
&c( 2. Patent ductus arteriosus is classically descri/ed in children as a continuous machinery-type murmur that
is widely transmitted across the precordium.
1>-. 2iagnostic 8tudies/Cardiology
A 6. year-old male with history o hypertension and to/acco a/use presents complaining o dyspnea on
e7ertion or two wee!s. The patient admits to one episode o chest discomort while sho"eling snow which was
relie"ed ater i"e minutes o rest. 4ital signs are 3P 1.,/>,) HR 6-) RR 1'. Heart e7am re"eals regular rate
and rhythm) normal 81 and 8$) no murmur) gallop) or ru/. Dungs are clear to auscultation /ilaterally. There is
no edema noted. 0hich o the ollowing is the most appropriate initial diagnostic study or this patient1
A. Helical CT scan
3. Chest 7-ray
C. +uclear stress test
2. Cardiac catheteri9ation
&u( A. Helical CT scan aids in the diagnosis o pulmonary em/olism) not in the e"aluation o angina.
&u( 3. Chest 7-ray is not used as a diagnostic study to e"aluate symptoms o angina or coronary heart disease.
&c( C. *n patients with classic symptoms o angina) nuclear stress testing is the most widely used test or
diagnosis o ischemic heart disease.
&u( 2. Coronary angiography is indicated in patients with classic sta/le angina who are se"erely symptomatic
despite medical therapy and are /eing considered or percutaneous inter"ention &PC*() patients with
trou/lesome symptoms that are diicult to diagnose) angina symptoms in a patient who has sur"i"ed sudden
cardiac death e"ent) patients with ischemia on nonin"asi"e testings.
1>B. Clinical *nter"ention/Cardiology
A %$ year-old male with history o hypertension and hyperlipidemia presents with an acute myocardial
inarction. Argent cardiac catheteri9ation is perormed and shows a B,I occlusion o the let anterior
descending artery. The other arteries ha"e minimal disease. C=ection raction is '%I. 0hich o the ollowing is
the treatment o choice in this patient1
A. Coronary artery /ypass grating &CA3<(
3. 8trepto!inase
C. Percutaneous coronary inter"ention &PC*(
2. 0ararin &Coumadin(
&u( A. Percutaneous coronary inter"ention is a /etter) less in"asi"e alternati"e to CA3< or single "essel
coronary artery disease.
&h( 3. 8trepto!inase is not commonly used or treatment o acute myocardial inarction /ecause it is ineecti"e
at opening the occluded artery and reducing mortality. 8trepto!inase would /e harmul /ecause it would
increase the ris! o /leeding.
&c( C. *mmediate coronary angiography and primary percutaneous coronary inter"ention ha"e /een shown to /e
superior to throm/olysis.
&u( 2. 0ararin is used to pre"ent throm/osis and not or acute treatment.
1-,. Clinical *nter"ention/;rthopedics/Rheumatology
An 1- year-old male presents with pain in his wrist ater he ell o o a mo"ing motor cycle. Physical
e7amination re"eals tenderness in the anatomic snu/o7. +o racture is noted on plain radiography o the wrist.
0hich o the ollowing is the recommended treatment or this patient1
A. Ace wrap o the wrist
3. Closed reduction o the racture site
C. Thum/ spica cast application
2. ;pen reduction o the racture site
&u( A) 3) 2. 8ee C or e7planation.
&c( C. C"en with normal initial radiographs) patients with a consistent history and tenderness in the anatomical
snu/o7 are treated as a sta/le racture with immo/ili9ation in a thum/ spica cast. Casting is recommended or
all presumed nondisplaced scaphoid ractures.
1-1. Clinical *nter"ention/Pulmonology
A solitary pulmonary nodule is ound on a pre-employment screening chest 7-ray in a .' year-old nonsmo!ing
male.There are no old chest 7-rays to compare. 0hich o the ollowing is the most appropriate ne7t step in the
e"aluation1
A. CT scan o the chest
3. +eedle /iopsy o the lesion
C. Positron emission tomography o the chest
2. Ei/eroptic /ronchoscopy
&c( A. *n the a/sence o old 7-rays in a nonsmo!ing indi"idual less than .% years old) CT scan o the chest is the
ne7t step in the e"aluation o a solitary pulmonary nodule.
&u( 3. A needle /iopsy would /e indicated or a person greater than .% years old and/or with a history o
smo!ing to e"aluate a solitary pulmonary nodule.
&u( C. Positron emission tomography &PCT scan( would /e indicated i the CT scan was nonconclusi"e.
&u( 2. Ei/eroptic /ronchoscopy would /e indicated only in the presence o a history o to/acco use or i the
lesion was suggesti"e o malignancy.
1-$. History : Physical/Psychiatry/3eha"ioral 6edicine
Carly clues to impending delirium tremens include
A. agitation and decreased cognition.
3. "isual hallucinations and diaphoresis.
C. autonomic hyperacti"ity and dehydration.
2. mental conusion and sensory hyperacuity.
&c( A. An7iety) decreased cognition) tremulousness) increasing irrita/ility) and hyperacti"ity are common early
clues to impending delirium tremens.
&u( 3. 6ental conusion) tremor) sensory hyperacuity) "isual hallucinations) autonomic hyperacti"ity)
diaphoresis) dehydration) electrolyte distur/ances) sei9ures) and cardio"ascular a/normalities are common signs
and/or symptoms o ull-/lown delirium tremens.
&u( C) 2. 8ee 3 or e7planation.
1-.. 8cientiic Concepts/C+T/;phthalmology
2ental caries are caused /y which o the ollowing organisms1
A. 8treptococcus mutans
3. 8treptococcus pyogenes
C. 8taphylococcus epidermidis
2. 8taphylococcus aureus
&c( A. 8treptococcus mutans is the principle organism that helps to deminerali9e the enamel.
&u( 3) C) 2. 8ee A or e7planation.
1-'. Clinical Therapeutics/Arology/Renal
The most deiniti"e treatment or primary enuresis is
A. o7y/utynin chloride &2itropan(.
3. imipramine &Toranil(.
C. trimethoprim-sulametho7a9ole &3actrim(.
2. desmopressin &22A4P(.
&u( A. ;7y/utynin chloride is used or /ladder spasms. *t cannot /e used or children under % years o age and is
not indicated in primary enuresis.
&u( 3. *mipramine is an older orm o treatment that is moderately eecti"e) /ut many patients relapse when
therapy is stopped. This is no longer considered the treatment o choice.
&u( C. T6P-86L is indicated or urinary tract inections that may cause secondary enuresis) /ut it is not used in
primary enuresis.
&c( 2. *ntranasal desmopressin is eecti"e in %,I o patients treated and is the treatment o choice.
1-%. 2iagnosis/;/stetrics/<ynecology
A '> year-old emale presents to the clinic with complaints o prolonged) hea"y menses that ha"e /een getting
progressi"ely worse or . years. 8he denies any pain. ;n physical e7amination) enlargement o the uterus with
multiple smooth) spherical) irm masses is noted. A C3C is consistent with a mild anemia. 0hich o the
ollowing is the most li!ely diagnosis1
A. Deiomyoma
3. Adenomyosis
C. Cndometriosis
2. Cndometrial polyps
&c( A. A/normal uterine /leeding and irregular enlargement o the uterus are most consistent with leiomyoma.
Pain is rarely present unless "ascular compromise occurs.
&u( 3. 0hile adenomyosis may present with hypermenorrhea) dysmenorrhea is oten also present. Physical
e7amination would re"eal the presence o diuse glo/ular uterine enlargement) not the irregular enlargement as
noted in the case presented.
&u( C. Cndometriosis presents with dyspareunia) dysmenorrhea) and inertility. * the pel"ic e7am were
a/normal) uterine indings would include tender nodules in the cul de sac) not the uterus.
&u( 2. 0hile endometrial polyps are compati/le with the history o a/normal uterine /leeding) the uterus would
/e normal si9e without the irregular enlargement noted in the case presented.
1-6. 2iagnostic 8tudies/<astrointestinal/+utritional
0hich o the ollowing is the la/oratory test that mar!s reco"ery rom Hepatitis 3 inection and non-
inecti"ity1
A. Hepatitis 3 surace anti/ody&anti-H3s(
3. Hepatitis 3 surace antigen &H3sAg(
C. Hepatitis 3 core antigen &H3cAg(
2. Hepatitis A anti/ody &anti-HA4(
&c( A. 8peciic anti/ody to H3sAg appears in most indi"iduals ater clearance o H3sAg which indicates
reco"ery rom hepatitis 3 inection) non-inecti"ity) and immunity.
&u( 3. This test esta/lishes inection with H34 and implies inecti"ity.
&u( C. Presence o the hepatitis 3 core antigen relects acti"e inection.
&u( 2. Patients who ha"e immunity to hepatitis A do not ha"e immunity to hepatitis 3.
1->. Health 6aintenance/Psychiatry/3eha"ioral 6edicine
A .6 year-old woman admits that her hus/and has a/used her or o"er ten years. Rou should inorm the woman
that she is at most ris! or in=ury or death
A. =ust /eore a holiday.
3. =ust ater lea"ing an a/usi"e spouse.
C. when an a/usi"e spouse arri"es home ater wor!.
2. when an a/usi"e spouse has /een drin!ing hea"ily.
&u( A) C) 2. 8ee 3 or e7planation.
&c( 3. 0omen are more li!ely to /e assaulted or murdered when attempting to report the a/use or lea"e the
a/usi"e relationshipF up to >%I o domestic assaults occur ater separation.
1--. Clinical *nter"ention/Cardiology
A patient presents with an acutely painul and cold let leg. 2istal pulses are a/sent. Deg is cyanotic. There are
no signs o gangrene or other open lesions. 8ymptoms occurred one hour ago. 0hich o the ollowing
treatments is most appropriate1
A. 4ena ca"a ilter
3. Cm/olectomy
C. Amputation
2. Aspirin
&u( A. 4ena ca"a ilters are used in the management o "enous throm/oem/olic disease when anticoagulation
cannot /e done.
&c( 3. Cm/olectomy within ' to 6 hours is the treatment o choice.
&h( C. Amputation is done only when no "ia/le tissue is present. Cutting o a "ia/le lim/ is ne"er a good idea.
&u( 2. Aspirin is used in the pre"ention and treatment o coronary disease and has no role in the treatment o
peripheral arterial em/olism.
1-B. 2iagnostic 8tudies/;rthopedics/Rheumatology
A %. year-old male is seen in the emergency department ollowing a motor "ehicle collision in which his !nee
impacted against the dash/oard. The patient has a posterior !nee dislocation that is promptly reduced in the
emergency department. The patient currently has a palpa/le pulse in the dorsalis pedis and posterior ti/ial areas.
0hich o the ollowing studies is mandatory1
A. Anterior plain ilm o !nee
3. 8unrise "iew o the !nee
C. 6easurement o compartment pressures
2. Angiography
&u( A) 3. 8ee 2 or e7planation.
&u( C. Compartment pressures are perormed in cases o suspected compartment syndrome) not to determine the
patency o the popliteal artery.
&c( 2. The popliteal artery is at ris! or in=ury whene"er a patient sustains a posterior dislocation o the !nee and
should /e e"aluated with an arteriogram despite the presence o pedal pulses.
1B,. 2iagnosis/Pulmonology
A $' year-old male presents complaining o a B month history o increasing shortness o /reath) dyspnea on
e7ertion) and a cough producti"e o white sputum) mostly in the mornings. He denies orthopnea) P+2)
peripheral edema) e"er) chills) night sweats) recent changes in weight) palpitations) chest pain) ood
intolerances) or other complaints. Patient has a history o recurrent lung inections. He states that his ather had
chronic pulmonary pro/lems and died at age '$ rom un!nown lung disease. The patient denies smo!ing)
alcohol or illicit drug use. ;n physical e7amination) the respiratory rate is $$ per minute) pulse o B- /pm)
temperature o B-.> degrees. Pulmonary e7am re"eals end-e7piratory whee9es /ilaterally and hyperresonance to
percussion. His cardiac e7am is normal. Chest 7-ray shows decreased lung mar!ings. CC< is normal.
Pulmonary unction tests show an EC41 6.I o e7pected and residual capacity is 1$.I o e7pected. 0hich o
the ollowing is the most li!ely diagnosis1
A. Cmphysema
3. Pulmonary i/rosis
C. 4entricular septal deect
2. Congesti"e heart ailure
&c( A. This person has an o/structi"e lung disease /ased on PETs. Cmphysema is the most li!ely d7) and may /e
related to alpha-1 antitrypsin deiciency /ased on amily history and lac! o smo!ing history and young age.
&u( 3. The PETs rom a person with pulmonary i/rosis would /e consistent with a restricti"e pattern. This
patient has an o/structi"e pattern o lung disease.
&u( C. 4entricular septal deect will ha"e a systolic murmur associated with it.
&u( 2. Congesti"e heart ailure might e7plain some o the symptoms o this patient &increasing shortness o
/reath and 2;C() he denies other common symptoms) such as orthopnea and peripheral edema. CHE should not
result in changes in the PETs.
1B1. 2iagnosis/Arology/Renal
A male patient complains o chronic dysuria) re@uency) and urgency with associated perineal pain. The most
li!ely diagnosis is
A. cystitis.
3. gonococcal urethritis.
C. epididymitis.
2. prostatitis.
&u( A. Cystitis is characteri9ed /y dysuria without urethral discharge.
&u( 3. *nitially there is /urning on urination and serous or mil!y discharge in gonococcal urethritis.
&u( C. Cpididymitis is characteri9ed /y dysuria) unilateral scrotal pain and swelling.
&c( 2. 8ome patients are asymptomatic) /ut low /ac! or perineal pain) e"er) chills) and irritati"e urinary
symptoms are common in prostatitis.
1B$. Health 6aintenance/;rthopedics/Rheumatology
0hich o the ollowing pre"enti"e strategies against osteoporosis-associated "erte/ral ractures has a !nown
side eect o increasing the incidence o hot lashes when used in a perimenopausal emale1 P
A. Calcitonin &6iacalcin( nasal spray
3. Alendronate &Eosama7(
C. Cstrogen/progesterone &Prempro(replacement
2. Ralo7iene &C"ista(
&u( A. Calcitonin does not ha"e any estrogen eects on the /ody and ser"es as an analgesic when used in the
management o "erte/ral ractures.
&u( 3. Alendronate is a /isphosphonate that does not ha"e any hormonal eects on the /ody.
&u( C. Cstrogen/progesterone replacement has the /eneit o maintaining /one and decreasing "erte/ral ractures
/ut it would impro"e perimenopausal hot lashes.
&c( 2. Ralo7iene has eects on /one turno"er and /one mass and has /een shown to decease "erte/ral
ractures. *t has anti-estrogen eects on the non-s!eletal portions o the /ody and increases hot lashes in
perimenopausal
emales.
1B.. Clinical Therapeutics/+eurology
A >, year-old presents with headache and nec! stiness. ;n physical e7am) the patient is e/rile) Kernig5s sign
is present) and no rash is noted. A spinal tap re"eals a white count o $%,/cm. with 1,,I neutrophils) total
protein $%, mg/dD) and glucose .% mg/dD. 0hich o the ollowing is the most appropriate treatment1
A. Acyclo"ir &Jo"ira7(
3. Elucona9ole &2ilucan(
C. Ampicillin and cetria7one &Rocephin(
2. Penicillin and chloramphenicol &Chloromycetin(
&u( A. Acyclo"ir is used to treat meningitis secondary to herpes. 4iral meningitis presents with increase
num/er o lymphocytes and ele"ated glucose in the C8E .
&u( 3. Elucona9ole is used to treat ungal meningitis. Eungal meningitis) typically noted in
immunocompromised hosts) presents with increase num/er o lymphocytes in the C8E.
&c( C. Ampicillin and cetria7one is used to treat /acterial meningitis) secondary to Disteria monocytogenes )
which is common in the elderly. Cetria7one will co"er other common etiologic agents such as 8treptococcal
pneumonia
&u( 2. Penicillin and chloramphenicol is used to treat /acterial meningitis) secondary to +eisseria meningitidis.
3acterial meningitis due to +. meningitis.
1B'. 2iagnostic 8tudies/Cndocrinology
A '% year-old patient with type 1 dia/etes mellitus is /eing screened or dia/etic nephropathy. 0hich o the
ollowing urinalysis indings is most consistent with early dia/etic nephropathy1
A. 6icroal/uminuria
3. Red cell casts
C. 0hite cell casts
2. Renal epithelial cells
&c( A. 6icroal/uminuria is most consistent with early dia/etic neuropathy.
&u( 3. Red cell casts are more indicati"e o acute glomerular nephritis.
&u( C. 0hite cell casts are more consistent with acute pyelonephritis.
&u( 2. A ew renal epithelial cells normally may /e ound in the urine.
1B%. 2iagnostic 8tudies/+eurology
A >' year-old emale is /eing treated or mild hypertension. 8he is ound at home with right hemiparesis and
/rought to the emergency department. Her daughter states that the patient ell in her !itchen $ days ago) /ut had
no complaints at that time. 8he did state that her mother sounded a little conused this morning. The patient5s
let pupil is dilated. 0hich o the ollowing diagnostic studies should /e ordered irst1
A. 6R* o the /rain
3. CT scan o the /rain
C. 8!ull 7-ray
2. Dum/ar puncture

&u( A. 8ee 3 or e7planation.
&c( 3. This patient presents with a history o minor trauma and progressi"e neurological a/normalities
consistent with su/dural hematoma. 2iagnosis would /e conirmed /y CT scan) which is less e7pensi"e and
more sensiti"e or /lood than an 6R*.
&u( C. 8!ull 7-rays would not /e helpul /ecause they e"aluate /ony) not sot tissue) in=ury.
&h( 2. A lum/ar puncture is contraindicated /ecause o the potential or /rain herniation.
1B6. 2iagnostic 8tudies/Pulmonology
A post-op patient has signs and symptoms highly suggesti"e o a pulmonary em/olism. The results o the CT
scan o the lung is nondiagnostic. 0hat is the most appropriate ne7t step in the e"aluation1
A. 4entilation perusion &4/H( scan
3. Altrasound o the legs
C. Cchocardiography
2. 2-dimer
&u( A. 4entilation perusion scans are perormed prior to the CT scan o the chest and would not li!ely add
additional inormation to this clinical scenario.
&c( 3. *n a patient with a high li!elihood o pulmonary em/olism or an inpatient) as in this case) ultrasound o
the legs would /e the ne7t diagnostic step ater a nondiagnostic CT.
&u( C. Although echocardiography may show right "entricular ree wall hypo!inesis with normal motion o the
ape7 suggesti"e o pulmonary em/olism) more than %,I o patients with a pulmonary em/olism will ha"e
normal echocardiography. Cchocardiography is not used in the diagnosis on inpatients.F
&u( 2. *n a post-op patient) a d-dimer will /e positi"e regardless o the presence or a/sence o a pulmonary
em/olism.
1B>. 2iagnosis/+eurology
8ei9ures that irst maniest in early to middle adult lie should /e considered suspicious o which o the
ollowing
causes1
A. Cere/ro"ascular disease
3. Cncephalitis
C. Tumor
2. *diopathic epilepsy
&u( A) 3) 2. 8ee C or e7planation.
&c( C. 8ei9ures that de"elop during adolescence and adult lie are predominantly due to tumor) trauma) drug use)
or alcohol withdrawal.
1B-. Clinical Therapeutics/Cardiology
0hich o the ollowing medications used in the treatment o supra"entricular tachycardia is a/le to cause sinus
arrest and asystole or a ew seconds while it /rea!s the paro7ysmal supra"entricular tachycardia1
A. 2igo7in &Dano7in(
3. Adenosine &Adenocard(
C. 4erapamil &Calan(
2. Huinidine &Huinaglute(
&u( A. 2igo7in is not used or the acute termination o supra"entricular tachycardia.
&c( 3. Adenosine is an endogenous nucleoside that results in proound &although transient( slowing o the A4
conduction and sinus node discharge rate. This agent has a "ery short hal-lie o 6 seconds.
&u( C. Although "erapamil may /e used or the termination o acute supra"entricular tachycardia) it does not
lead to sinus arrest in therapeutic doses.
&u( 2. Huinidine is rarely used today and is not indicated or the termination o supra"entricular tachycardia.
Re# &1%(
1BB. Clinical *nter"ention/<astrointestinal/+utritional
A .$ year-old presents with a .-day history o diarrhea. The patient denies /lood) mucus) or night awa!ening
with diarrhea. He recently returned rom a /usiness trip to Canada. ;n physical e7amination) the patient is
ae/rile and "ital signs re"eal 3P 11%/-,) pulse is >6) and respirations are 1'. The a/dominal e7amination
re"eals hyperacti"e /owel sounds) /ut is otherwise unremar!a/le. 0hich o the ollowing is the most
appropriate initial inter"ention1
A. 8tool or culture) o"a) and parasites
3. Proctosigmoidoscopy
C. 6etronida9ole &Elagyl(
2. 8upporti"e treatment
&u( A. 8tool culture and e7amination or o"a and parasites are indicated when the diarrhea has persisted longer
than . wee!s or is associated with a/dominal pain) e"er) and/or /loody stools.
&u( 3. Proctosigmoidoscopy is indicated when inlammatory /owel disease is suspected on the /asis o e"er)
/loody diarrhea) or a/dominal pain.
&u( C. 6etronida9ole is indicated with a conirmed diagnosis o <iardia lam/lia or ame/ic disease.
&c( 2. 8ymptomatic treatment) including dietary management and o"er-the-counter antidiarrheals) is indicated
or ae/rile patients with watery diarrhea o less than % days duration.
$,,. 2iagnostic 8tudies/2ermatology
A $6 year-old emale presents with se"eral pruritic lesions on her dorsal orearms. The lesions are erythematous
with "esicles) with a ew /eginning to weep. 8he wor!s in a photography la/oratory) /ut denies any other
possi/le e7posures. 0hich o the ollowing is the most useul diagnostic test1
A. 42RD serology
3. K;H prep
C. Patch testing
2. <ram5s stain
&u( A. 42RD serology is useul in the diagnosis o syphilis) not contact dermatitis.
&u( 3. K;H prep is used or diagnosis o ungal inections) not contact dermatitis.
&c( C. Patch testing with a suspected agent is usually positi"e in cases o allergic contact dermatitis.
&u( 2. <ram5s stain is useul in the diagnosis o /acterial inections) not contact dermatitis.
$,1. 2iagnosis/Cardiology
An elderly emale presents or e"aluation o e7ertional syncope) dyspnea) and angina. 8he admits that pre"ious
to these symptoms she had insidious progression o atigue that caused her to curtail her acti"ities. 0hich o the
ollowing is the most li!ely diagnosis1
A. Aortic stenosis
3. Aortic regurgitation
C. 6itral stenosis
2. 6itral "al"e prolapse
&c( A. The ma=or symptoms o aortic stenosis are e7ertional syncope) dyspnea) and angina. 8ymptoms do not
/ecome apparent or a num/er o years and usually are not present until the "al"e is narrowed to less than
,.%cm to $ cm o "al"e surace area.
&u( 3. Patients with aortic regurgitation are li!ely to complain o an uncomorta/le awareness o their heart)
especially when lying down. These patients de"elop sinus tachycardia with e7ertion and complain o
palpitations and head pounding with acti"ity.
&u( C. The symptoms related to mitral stenosis are related to increased pulmonary pressure ater the let atrium
can no longer o"ercome the outlow o/struction.
&u( 2. Patients with mitral "al"e prolapse are typically asymptomatic throughout their li"es) although a wide
range o symptoms is possi/le. 0hen symptoms do occur) palpitations rom arrhythmias are most common
along with lightheadedness. 8yncope is not part o this disease process.
$,$. Clinical *nter"ention/Arology/Renal
A ', year-old emale <%P% complains o small @uantities o urine lea!ing when she coughs) snee9es) or laughs.
Her genitourinary e7amination is unremar!a/le and her urinalysis is normal. At this time) which o the
ollowing is the most appropriate management plan1 P
A. Reer or a cystoscopy.
3. Recommend Kegel e7ercises.
C. Reer or surgical correction.
2. Recommend hormone replacement therapy.
&u( A. Conser"ati"e therapy or stress incontinence should /e attempted prior to any e"aluation) such as
cystoscopy) that might indicate the need or surgical correction.
&c( 3. 8trengthening the pel"ic muscles /y Kegel e7ercises and emptying the /ladder re@uently may resol"e the
pro/lem.
&u( C. 8ee A or e7planation.
&u( 2. There is no indication in the history or hormone replacement therapy and no "aginal atrophy was noted
on pel"ic e7amination.
$,.. 2iagnosis/<astrointestinal/+utritional
A 6$ year-old male presents with complaints o "ague epigastric a/dominal pain associated with =aundice and
generali9ed pruritus. Physical e7amination re"eals =aundice and a palpa/le non-tender gall/ladder) /ut is
otherwise unremar!a/le. 0hich o the ollowing is the most li!ely diagnosis1
A. 4iral hepatitis
3. Pancreatic cancer
C. Acute cholecystitis
2. <il/ert5s syndrome
&u( A. 0hile "iral hepatitis may cause =aundice) the li"er is enlarged and tender.
&c( 3. Pancreatic cancer is suggested /y the "ague epigastric pain with the =aundice resulting rom /iliary
o/struction due to cancer in"ol"ing the pancreatic head. The presence o a palpa/le non-tender gall/ladder
&Cour"oisier5s sign( also indicates o/struction due to the cancer.
&u( C. 0hile acute cholecystitis may present with =aundice and an enlarged gall/ladder) the pain is classically
colic!y and located in the right upper @uadrant. ;n physical e7amination with deep inspiration and palpation o
the right su/costal area increased pain and respiratory arrest &6urphy5s sign( is usually seen.
&u( 2. <il/ert5s syndrome is the most common o the hereditary hyper/iliru/inemias. *t is most oten diagnosed
near pu/erty or adult lie /ased on results o a comprehensi"e meta/olic panel.
$,'. History : Physical/Cardiology
0hich o the ollowing would you e7pect on physical e7amination in a patient with mitral "al"e stenosis1
A. 8ystolic /lowing murmur
3. ;pening snap
C. 6id-systolic clic!
2. Parado7ically split 8$
&u( A. 6itral stenosis is a diastolic) not a systolic murmur.
&c( 3. 6itral stenosis is characteri9ed /y a mid-diastolic opening snap.
&u( C. 6id-systolic clic!s are noted in mitral "al"e prolapse) not mitral stenosis.
&u( 2. Parado7ical splitting o 8$ occurs in aortic stenosis not mitral stenosis.
$,%. 2iagnostic 8tudies/2ermatology
A positi"e 0ood5s light e7amination &luorescence( demonstrates
A. "iral inection with herpes 9oster.
3. /acterial inection with Treponema pallidum.
C. parasitic inestation with Pediculus humanus.
2. mycotic inection with 6icrosporum canis.
&u( A. Desions o herpes) Treponema) and Pediculus inections do not luoresce.
&u( 3) C. 8ee A or e7planation.
&c( 2. 6icrosporum causes tinea capitis and luoresces /lue-green under 0ood5s light.
$,6. History : Physical/2ermatology
*n which o the ollowing patients would one most li!ely ind acanthosis nigricans1
A. A %% year-old o/ese emale with hyperinsulinemia
3. A %% year-old male with an enlarged spleen and pancytopenia
C. A $' year-old emale with increased Dyme titers
2. A 6, year-old male with increased triglycerides
C7planations
&c( A. Acanthosis nigricans is associated with patients who ha"e hyperinsulinemia.
&u( 3. Hairy cell leu!emia is associated with an enlarged spleen and pancytopenia. Acanthosis nigricans is not a
cutaneous maniestation.
&u( C. Crythema chronicum migrans is the characteristic lesion associated with Dyme disease.
&u( 2. Crupti"e 7anthelasma is associated with increased triglycerides.
$,>. History : Physical/;rthopedics/Rheumatology
A 6% year-old male presents with /ac! pain two days ater he was sho"eling snow. The patient complains o
pain in his low /ac! that radiates into his /uttoc!s) posterior thigh and cal) and the /ottom o his oot. There is
associated num/ness o his lateral and plantar surace o his oot. 0hich o the ollowing disc herniations is
most li!ely to /e aected1
A. D.-D'
3. D'-D%
C. D%-81
2. 81-8$
&u( A) 3) 2. 8ee C or e7planation.
&c( C. The 81 ner"e root impingement is most li!ely to occur rom the herniation o the D%-81 disc space. The
81 disc aects Achilles5 rele7) the gastrocnemius and soleus muscles) and the a/ductor hallucis and gluteus
ma7imus muscles.
$,-. Clinical *nter"ention/Psychiatry/3eha"ioral 6edicine
A $$ year-old emale presents to the emergency department with rapid heart rate. 8he appears @uite thin and
dehydrated. 8he denies that she is thin) stating G* am so at that * can hardly stand myselS That is why * e7ercise
e"ery day.G 8he runs twel"e to iteen miles a day) and on wee!ends also /icycles orty to ity miles. Her D6P
was si7 months ago. ;n e7am) she is %5 6G tall and weighs B- pounds. Temp B- degrees E) pulse 1,,)
respirations 1-) 3P B-/6,. CK< shows sinus tachycardia. Da/oratory indings include +a 1.- mC@/D) K $.-
mC@/D) Cl B1 mC@/D) 3A+ .% mg/dD) Creatinine 1.1 mg/dD. 0hich o the ollowing is the ne7t most
appropriate treatment1
A. Propylthiouracil &PTA( and indi"idual psychotherapy
3. Hormone replacement and cogniti"e therapy
C. 0eight restoration and amily therapy
2. *4 hydration and antidepressant therapy
&u( A. Propylthiouracil is used to treat hyperthyroidism) not anore7ia ner"osa.
&u( 3. Hormone replacement is not indicated or the amenorrhea o anore7ia ner"osa) /ut nutritional support
may help.
&c( C. Anore7ia ner"osa re@uires a comprehensi"e) multidisciplinary approach to treatment that integrates
medical management) indi"idual psychotherapy) and amily therapy. Currently) the /est results ha"e /een
shown with weight restoration accompanied /y amily therapy or patients with adolescent-onset anore7ia
ner"osa and indi"idual therapy or patients with onset ater 1- years o age. *npatient treatment is oten
re@uired.
&u( 2. Antidepressant therapy may /e useul in /ulimia ner"osa.
$,B. 2iagnosis/C+T/;phthalmology
A %$ year-old emale presents with complaints o intermittent episodes o di99iness) tinnitus) and hearing loss in
the right ear or 6 months. 8he descri/es the di99iness as the Groom spinning around her)G with the episodes
typically lasting or $ to ' hours. Physical e7amination re"eals hori9ontal nystagmus and right ear hearing loss)
/ut the remainder o the e7amination is unremar!a/le. 0hich o the ollowing is the most li!ely diagnosis1
A. Acute la/yrinthitis
3. Positional "ertigo
C. Acoustic neuroma
2. 6TniUre5s syndrome
&u( A. Acute la/yrinthitis typically presents with an acute onset o continuous "ertigo that lasts se"eral days to a
wee! and is associated with nausea and "omiting. *t does not ha"e any associated auditory or neurologic s7s.
&u( 3. Positional "ertigo occurs ollowing changes in head positioning with "ery /rie) less than 1 minute)
episodes. +ystagmus occurs ollowing the position change.
&u( C. Acoustic neuroma typically presents with hearing loss and tinnitus. The neuroma grows slowly and
central compensatory mechanisms can pre"ent or minimi9e the "ertigo. 4ertigo) when present) is continuous
and not episodic.
&c( 2. 6TniUre5s syndrome usually presents with episodes o "ertigo that last rom 1 to - hours) sensorineural
hearing loss and tinnitus.
$1,. 8cientiic Concepts/Cardiology
0hich o the ollowing is the most common cause or acute myocardial inarction1
A. ;cclusion caused /y coronary microem/oli
3. Throm/us de"elopment at a site o "ascular in=ury
C. Congenital a/normalities
2. 8e"ere coronary artery spasm
&u( A. Coronary microem/oli occlusion is a rare cause o acute myocardial inarction.
&c( 3. Acute myocardial inarction occurs when a coronary artery throm/us de"elops rapidly at a site o
"ascular in=ury. *n most cases) inarction occurs when an atherosclerotic pla@ue issures) ruptures) or ulcerates
and when conditions a"or throm/ogenesis) so that a mural throm/us orms at the site o rupture and leads to
coronary artery occlusion.
&u( C. Congenital a/normalities are rare causes o acute 6*.
&u( 2. 8e"ere coronary artery spasm is more li!ely to result in Prin9metal5s angina rather than true inarction.
$11. Health 6aintenance/;rthopedics/Rheumatology
Cndotracheal intu/ation should /e perormed with caution in patients with which o the ollowing underlying
conditions due to the propensity to cause su/lu7ation o C1 on C$1
A. Rheumatoid arthritis
3. ;steoarthritis
C. <out
2. Pseudogout
&c( A. Patients with ad"anced rheumatoid arthritis will ha"e syno"itis o the atlantoa7ial =oint &C1-C$( which
may damage the trans"erse ligament o the atlas) producing orward displacement o the atlas on the a7is
&atlantoa7ial su/lu7ation(.
&u( 3. Although patients with osteoarthritis may ha"e nec! pain and stiness) there is no predilection or the
atlantoa7ial =oints.
&u( C. Patients with gout are li!ely to ha"e in"ol"ement o peripheral =oints rather than spinal =oints.
&u( 2. Patients with pseudogout are more li!ely to ha"e in"ol"ement o the !nees) wrist) shoulder) an!le) el/ow)
and hands rather than the cer"ical spine.
$1$. Clinical *nter"ention/2ermatology
0hich o the ollowing inter"entions is the treatment o choice or actinic !eratosis1
A. 6ohs surgery
3. Cryotherapy
C. Acid peels
2. Radiation therapy
&u( A. 6ohs surgery and radiation therapy are not indicated in the treatment or actinic !eratosis.
&c( 3. Cryotherapy is the treatment o choice or isolated supericial actinic !eratosis.
&u( C. Acid peels can /e used to treat actinic !eratosis /ut are not the treatment o choice.
&u( 2. 8ee A or e7planation.
$1.. History : Physical/;rthopedics/Rheumatology
A %% year-old secretary presents with ongoing pain and num/ness in her hand. These symptoms are worse at
night and she must sha!e her hand to regain eeling in it. 0hich o the ollowing physical e7amination signs
will /e present1
A. Hypothenar atrophy
3. 0ea!ness o inger a/duction
C. *na/ility to maintain wrist e7tension against resistance
2. 0ea!ness o thum/ a/duction
&u( A. Hypothenar atrophy may occur with aging and disuse /ut it is not part o the median ner"e in"ol"ement
that occurs with carpal tunnel syndrome.
&u( 3. Einger a/duction wea!ness is associated with ulnar ner"e in=ury) which does not occur with carpal tunnel
syndrome.
&u( C. Radial ner"e in=ury causes wea!ness o wrist e7tension and this is not part o carpal tunnel syndrome.
&c( 2. 6edian ner"e in=ury causes wea!ness o thum/ a/duction &measured /y thum/ opposition strength( along
with thenar atrophy. Tinel5s and Phalen5s signs will also /e positi"e with carpal tunnel syndrome.
$1'. Health 6aintenance/Cardiology
A >- year-old male with history o coronary artery disease status post CA3< and ischemic cardiomyopathy
presents with complaint o progressi"e dyspnea and orthopnea. He also complains o lower e7tremity edema.
The patient denies e"er) chest pain) or cough. ;n physical e7amination) "ital signs are 3P 1$,/6-) HR >% and
regular) RR $$) ae/rile. Rou note the patient to ha"e an 8. heart sound) =ugular "enous distention) and $V
lower e7tremity edema. The patient is admitted and treated. Apon discharge rom the hospital) the patient
should /e educated to monitor which o the ollowing at home1
A. 2aily weights
3. 2aily spirometry
C. 2aily /lood glucose
2. 2aily at inta!e
&c( A. Home monitoring o daily weights can alert the health care pro"ider to the early recognition o worsening
heart ailure.
&u( 3. 8pirometry monitoring is important in a patient with asthma) not heart ailure.
&u( C. 2aily /lood glucose monitoring is important in a patient with dia/etes) not heart ailure.
&u( 2. 2aily at inta!e is important) /ut will not impro"e his heart ailure management.
$1%. 8cientiic Concepts/;rthopedics/Rheumatology
0hich o the ollowing rotator cu tendons is most li!ely to sustain in=ury /ecause o its repeated impingement
&impingement syndrome( /etween the humeral head and the undersurace o the anterior third o the acromion
and coracoacromial ligament1
A. 8upraspinatus
3. *nraspinatus
C. Teres minor
2. 8u/scapularis
&c( A. A critical 9one e7ists or the supraspinatus tendon due to its superior insertion site. *t is suscepti/le or
in=ury /ecause it has a reduction in its /lood supply that occurs with a/duction o the arm. *mpingement o the
shoulder is most commonly seen with the supraspinatus tendon) the long head o the /iceps tendon and/or the
su/acromial /ursa.
&u( 3) C) 2. 8ee A or e7planation.
$16. 2iagnostic 8tudies/Arology/Renal
0hich o the ollowing diagnostic indings in the urinary sediment is speciic or a diagnosis o chronic renal
ailure1
A. Hematuria
3. Proteinuria
C. 3road wa7y casts
2. Hyaline casts
&u( A. Hematuria and proteinuria are re@uent) /ut nonspeciic) indings in chronic renal ailure.
&u( 3. 8ee A or e7planation.
&c( C. 3road wa7y casts in urinary sediment are a speciic inding in chronic renal ailure.
&u( 2. Hyaline casts may /e ound in normal urine or in states o dehydration.
$1>. Health 6aintenance/C+T/;phthalmology
*n addition to to/acco products) which o the ollowing is also considered a ma=or ris! actor in the de"elopment
o oral cancer1
A. 8un e7posure
3. Alcohol a/use
C. ;ccupational e7posure
2. History o oral candidiasis
&u( A. 8un e7posure is a ris! actor or cancer o the lip) /ut is not considered a ma=or ris! actor or oral cancer.
&c( 3. 6a=or ris! actors or de"elopment o oral cancer are use o to/acco products and alcohol a/use.
&u( C. 0hile occupational e7posures and presence o premalignant lesions) such as leu!opla!ia) are ris! actors
or de"elopment o oral cancer) they are not considered ma=or ris! actors.
&u( 2. History o oral candidiasis has no correlation to de"elopment o oral cancer.
$1-. Clinical *nter"ention/C+T/;phthalmology
A 1. year-old /oy with leu!emia presents with epista7is or $ hours. The /leeding site appears to /e rom
Kiessel/ach5s area. The most appropriate inter"ention is
A. electrocautery o the /leeding site.
3. sil"er nitrate application.
C. posterior nasal pac!ing.
2. intranasal petrolatum gau9e.
&u( A. Cautery is not used /ecause the edges o the cauteri9ed area may /egin to /leed.
&u( 3. 8il"er nitrate is not used in children /ecause it increases the ris! or nasal septal peroration.
&u( C. Posterior nasal pac!ing is indicated or posterior /leeds in the inerior meatus.
&c( 2. Petrolatum gau9e will pro"ide pressure to the /leeding point while the cause o /leeding is corrected.
$1B. Clinical Therapeutics/C+T/;phthalmology
A $ year-old emale presents with purulent nasal discharge /ilaterally with e"er and cough or se"eral days.
Her mom had ta!en her out o daycare or a similar occurrence $ months ago) that was treated with Amo7icillin.
C7am urther re"eals halitosis and perior/ital edema. Treatment should /e initiated with which o the
ollowing1
A. Antihistamines
3. Ri/a"irin &Re/etol(
C. *ntranasal corticosteroids
2. Amo7icillin-cla"ulanate &Augmentin(
&u( A. Antihistamines and intranasal corticosteroids ha"e not /een ade@uately studied in children to pro"e they
ma!e a dierence in treating recurrent sinusitis.
&u( 3. Ri/a"irin is appro"ed or the treatment o R84 inection.
&u( C. 8ee A or e7planation.
&c( 2. High dose amo7icillin-cla"ulanate is the treatment o choice or resistant /acterial sinusitis) especially in
children presenting with ris! actors &daycare attendance) pre"ious anti/iotic treatment 1-. months prior) age
younger than $ years(.
$$,. Health 6aintenance/*nectious 2iseases
0hich o the ollowing otherwise healthy persons should recei"e annual inluen9a immuni9ation1
A. 1% year-old high school student
3. $. year-old physician assistant student
C. .$ year-old seminary student
2. '- year-old doctoral &history( student
&u( A) C) 2. 8ee 3 or e7planation.
&c( 3. Adults o"er 6%) and adults and children who are at high ris! or contracting inluen9a or suering
complications should /e immuni9ed. Health care pro"iders) including health care students who come in contact
with patients with inluen9a and who can /ecome carriers and inect others should /e immuni9ed.
$$1. 8cientiic Concepts/Cardiology
0hich o the ollowing is the most common cause o arterial em/oli9ation1
A. Rheumatic heart disease
3. 6y7oma
C. Atrial i/rillation
2. 4enous throm/osis
&u( A. Rheumatic heart disease is a rare cause o em/oli9ation
&u( 3. 6y7oma is a rare cause o em/oli9ation.
&c( C. Atrial i/rillation is present in 6,->,I o patients with arterial em/oli and is associated with let atrial
appendage throm/us.
&u( 2. 4enous throm/osis may /e a cause o em/oli9ation parado7ically) /ut is uncommon.
$$$. 2iagnosis/<astrointestinal/+utritional
A 16 year-old male is /rought into your oice /y his girlriend. 8he states that Ghe hasn5t /een himsel latelyG
and seems to luctuate rom /eing almost GeuphoricG to /eing depressed and irrita/le. The patient states that Ghe
is really o!ayG and that he G=ust eels a little irrita/le occasionally.G ;n physical e7amination his pulse is 1$,
/eats/minute) /lood pressure is 1-,/11, mmHg) he is sweating and his pupils are widely dilated. 0hich o the
ollowing is the most li!ely diagnosis1
A. ;piate a/use
3. Acute an7iety attac!
C. Cocaine into7ication
2. 3ipolar aecti"e disorder
&u( A. ;piate a/use would /e suspected with the presence o euphoria) drowsiness) and constricted pupils.
6ore se"ere cases present with /radycardia) hypotension) coma) or respiratory arrest.
&u( 3. 0hile acute an7iety disorder may present with tachycardia) agitation) diaphoresis) and hypertension) this
diagnosis should /e a diagnosis o e7clusion. Eailure to diagnose cocaine into7ication could lead to se"ere
conse@uences.
&c( C. Cocaine is a stimulant and presenting clinical maniestations o into7ication include agitation)
tachycardia) hypertension) diaphoresis) and dilated pupils.
&u( 2. 0hile the history may suggest the diagnosis o /ipolar aecti"e disorder) the physical e7am indings do
not support this diagnosis.
$$.. Clinical Therapeutics/<astrointestinal/+utritional
A $% year-old man presents with odynophagia and dysphagia. ;n endoscopic e7amination) small) white)
patches with surrounding erythema are noted. 8il"er stain is positi"e or hyphae. The /est treatment option or
this patient is
A. acyclo"ir &Jo"ira7(.
3. omepra9ole &Prilosec(.
C. lucona9ole &2ilucan(.
2. penicillin <.
&u( A. Acyclo"ir is an anti"iral used in the treatment o herpes esophagitis.
&u( 3. ;mepra9ole is a proton pump inhi/itor used in the treatment o gastroesophageal relu7 disease with
esophageal ulceration and peptic ulcer disease and is not indicated in the treatment o inectious esophagitis.
&c( C. The patient has Candida esophagitis and the treatment o choice is lucona9ole.
&u( 2. Penicillin < is an anti/iotic and is not eecti"e against ungal inections.
$$'. 8cientiic Concepts/Cardiology
The most common arrhythmia encountered in patients with mitral stenosis is
A. atrial lutter.
3. atrial i/rillation.
C. paro7ysmal atrial tachycardia.
2. atrio-"entricular dissociation.
&u( A) C) 2. 8ee 3 or e7planation.
&c( 3. 6itral stenosis leads to enlargement o the let atrium) which is the ma=or predisposing ris! actor or the
de"elopment o atrial i/rillation.
$$%. 2iagnostic 8tudies/Psychiatry/3eha"ioral 6edicine
A mother /rings her teenage daughter to the emergency department. The teenager is an7ious) tremulous) and in
a dysphoric mood. 8he reports recent nightmares and insatia/le hunger. 0hich o the ollowing diagnostic tests
would li!ely yield the most important inormation1
A. 8erum T8H
3. Arine drug screen
C. 6innesota 6ultiphasic Personality *n"entory &66P*(
2. Clectroencephalogram &CC<(
&u( A. 0hile hyperthyroidism can cause tremor and sometimes hunger) it is not associated with a dysphoric
mood. *t also generally occurs in early adulthood.
&c( 3. A drug screen would /e helpul or many drugs o a/use that might cause these symptoms) /ut may not
/e deiniti"e or amphetamines.
&u( C. The 66P* is an o/=ecti"e personality assessment instrument. There is no indication or its use here.
&u( 2. 0hile an CC< might /e useul later &assuming the drug screen was negati"e() there is no history to
indicate use o this test early on.

Das könnte Ihnen auch gefallen